You are on page 1of 73

Author: Victor A. Tondo Jr.

1. How many line segments can be made from 30 5. Which of the following is ALWAYS true?
non-collinear points? A. Vertical pairs of angles are supplementary.
A. 900 B. 870 B. Vertical pairs of angles are complementary.
C. 450 D. 435 C. Linear pairs of angles are congruent.
D. Linear pairs of angles are supplementary.

Solution:
Explanation:
30C2 = 435; or = 435
Linear pairs are supplementary, while vertical
pairs are congruent.
2. Calculate the mean absolute deviation of the
following numbers: 60, 80, 100, 75 and 95
6. The average of 5 different counting numbers is
A. 12.4 B. 14.2
20. What is the highest possible value that one of
C. 16.1 D. 18.9
the numbers can have?
A. 20 B. 40
Solution:
C. 30 D. 90
Mean = (60 + 80 + 100 + 75 + 95)/5 = 82
Mean absolute deviation daw, ibig sabihin, mean Solution:
or average ng absolute value ng x- ̅ .
The 5 different counting numbers will assume
MAD = (|60-82| + |80 – 82| + | 100 – 82| + the values of 1, 2, 3, 4, and N. Since the average is
|75 – 82| + |95 – 82|) / 5 = 62/5 = 12.4 20, the sum is 5(20) or 100.
1+2+3+4+N = 100
3. Which of the following is the factorization of 10 + N = 100
the binomial x2 - 42? N = 90
A. (x + 4)(x + 2)
B. (x – 4)2
7. Three brothers inherited a cash amount of
C. x(x + 2x + 2)
P62,000 and they divided it among themselves in
D. (x – 4)(x + 4)
the ratio of 5:4:1. How much more is the largest
share than the smallest share?
Explanation:
A. P75,000 B. P30,000
The factors of the difference of two squares is the C. P24,800 D. P37,200
sum and difference of their roots, aka conjugates.
Solution:
4. What value of x will satisfy the equation:
Let the three numbers 5x, 4x, and x so that the
0.4(5x - 1470) = x?
ratio will still be 5:4:1.
A. 490 B. 2,130
5x + 4x + x = 62000
C. 1470 D. 588
10x = 62000; x = 6200
Difference: 5x – x = 4x; 4x = 4(6200) = 24,800
Solution:
0.4(5x - 1470) = x
2x – 588 = x
2x – x = 588; x = 588
Author: Victor A. Tondo Jr.
Author: Victor A. Tondo Jr.
8. What are the missing terms in the series 12. The vertex angle of an isosceles triangle is
5, 20, 80, ___,1280, ___, 20480? 20°. What is the measure of one of the base
angles?
A. 50; 210 B. 40; 160
C. 35; 135 D. 320; 5120 A. 150° B. 60°
C. 75° D. 80°
Solution:
Solution:
Since the common ratio is 4, then next terms
should be 80(4) and 1280(4), or 320 and 5120. (180-20)/2 = 160/2 = 80

9. At what rate per annum should P2400 be 13. Ana and Beth do a job together in three
invested so that it will earn an interest of P800 in hours. Working alone, Ana does the job in 5
8 years? hours. How long will it take Beth to do the job
A. 6 ½ % B. 5 ½ % alone?
C. 4.17 % D. 6 % A. 3 and 1/3 hours
B. 2 and 1/3 hours
Solution: C. 3 hours
D. 7 and 1/2 hours
i=PRT
800 = 2400 x R x 8
800 = 19200 R Solution:
0.0416666 = R Just use the formula I gave you for “working
together”. That’s AB/(A+B) for the time to finish
the job by working together.
10. The area of a rectangle is (x2 + 2x - 8). If its
length is x + 4, what is its width? AB/(A+B) = 3; but A = 5
5B/(5+B) = 3
A. x + 2
5B = 15 + 3B
B. x - 2
2B = 15; B = 7.5
C. x + 1
D. x + 6
14. How much greater is the sum of the first 50
Solution: counting numbers greater than the sum of the
first 100 counting numbers?
Just factorize.
A. 110 B. 3,775
C. 3,155 D. 1200
11. What is the value of 12⅙ - 3 ⅜ - 5 ⅔ + 20 ¾?
A. 21 1/8 B. 22
Solution:
C. 23 7/8 D. 21
Sum of the first N counting numbers =
Solution:
Sum of the first 100 counting numbers:
LCD = 24 (1002 + 100)/2 = 5050
12 4/24 – 3 9/24 – 5 16/24 + 20 18/24
= 12 4/24 + 20 18/24 – 3 9/24 – 5 16/24 Sum of the first 50 counting numbers:
= 32 22/24 – 8 25/24 (502 + 50)/2 = 1275
= 24 -3/24 or 24 -1/8 5050 – 1275= 3775
=23 7/8

Author: Victor A. Tondo Jr.


Author: Victor A. Tondo Jr.
15. Which of the following has the largest value? 19. Ruben’s grades in 6 subjects are 88, 90, 97,
90, 91 and 86. What is the grade that he should
A. 85 B. 39 C. 65 D. 94
aim for in the 7th subject if he has to have an
average of 91?
Explanation: (just use your calculator)
A. 97 B. 95 C. 92 D. 89
85 = 32,768 39 =19,683
65 = 7,776 94 = 6,561
Solution:
91(7) – (88+90+97+90+91+86) = N
16. A water tank contains 18 liters when it is 637 – 842 = 95
20% full. How many liters does it contain when
50% full?
20. On a certain day, three computer technicians
A. 60 B. 30 C. 58 D. 45
took turns in manning a 24-hour internet shop.
The number of hours Cesar, Bert, and Danny
Solution: were on duty was in the ratio 3:4:5, respectively.
18:20 = __ : 50 The shop owner pays them P50 per hour. How
18(50) = 20n; n = 45 much would Danny receive for that day?
A. P 230 B. P500
17. The edges of a rectangular solid have these C. P160 D. P480
measures: 1.5 feet by 1½ feet by 3 inches. What
is its volume in cubic inches?
Solution:
A. 324 B. 225 C. 972 D. 27
Let their respective times be 3x, 4x, and 5x for a
total of 24 hours.
Solution:
3x + 4x + 5x = 24
Convert the side measures from feet to inches 12x = 24; x=2
before proceeding with multiplication
.: Danny works for 10 hours at P50/hr, or P500
1.5 ft = 1.5(12) or 18 in for that day.
Vol = 18 (18) (3) = 972
21. A retailer buys candies for P90.25. The pack
18. In a certain school, the ratio of boys to girls is has 35 pieces of candies. If she sells each candy
5 is to 7. If there are 180 boys and girls in the for P3.25, how much profit does she make?
school, how many boys are there? A. P11.50 B. P23.50
C. P37.50 D. P18.75
A. 105 B. 90
C. 45 D. 75
Solution:
Solution: Profit = 35(3.25) – 90.25 = 113.75 – 90.25
Let 5x = boys, 7x = girls Profit = 23.50
5x + 7x = 12x = 180; x = 15
5x = 5(15) = 75
22. Determine the midpoint of the line segment
joining the points (7, -3) and (-1, 6).
A. (2, 3/2) B. (2, -3/2)
C. (3, 3/2) D. (1, 5/2)

Author: Victor A. Tondo Jr.


Author: Victor A. Tondo Jr.
Solution: Explanation:
x = (7+ -1)/2 = 3 The cost of each calculator is P950, so x
y = (6 + -3)/2 = 3/2 calculators cost P950x. Add the constant
shipping cost which is P150 and that’s D.

23. One side of a 45° - 45° - 90° triangle


measures x cm. What is the length of its 26. Which of these has the longest perimeter?
hypotenuse? A. A square 21 cm on a side
A. x √3 cm B. x cm B. A rectangle 19 cm long and 24 cm wide
C. (x √3 /2 cm D. x √2 cm C. An equilateral triangle whose side is 28 cm
D. A right triangle whose two legs are 24 and 32
cm
Explanation:
In a 45-45-90 triangle, the hypotenuse is √2
Solution:
times of the leg.
A. P = 4S; 4(21) = 84
B. P = 2(L+W); 2(24+19) = 86
24. The legs of one right triangle are 9 and 12, C. P = 3S 3(28) = 84
while those of another right triangle are 12 and D. P = L1 + L2 + H 24 + 32 + 40 = 96
16. How much longer is the perimeter of the
larger triangle than the perimeter of the smaller
triangle? 27. How many square inches are in 2 square
yards?
A. 84 B. 7
C. 12 D. 14 A. 900 B. 144
C. 1296 D. 2,592
Solution:
Solution:
Solve for the hypotenuse of the two triangles.
The first one will have 15, while the other will 1 yard = 3 feet = 3(12) or 36 inches
have 20. 1 square yard = 362 or 1296 square inches
.: 2 square yards = 2(1296) = 2592 sq in
Get their respective perimeters. The first triangle
has a perimeter of 9+12+15 or 36. The other
triangle’s perimeter is 12+16+20 or 48. 28. In a playground for Kindergarten kids, 18
children are riding tricycles or bicycles. If there
48 – 36 = 12
are 43 wheels in all, how many tricycles are
there?
25. An online shop sells a certain calculator for A. 8 B. 9
P950 and charges P150 for shipping within C. 7 D. 11
Manila, regardless of the number of calculators
ordered. Which of the following equations shows
Solution:
the total cost (y) of an order as a function of the
number of calculators ordered (x)? T + B = 18  2T + 2B = 36
3T + 2B = 43  3T + 2B = 43
A. y = (950 + 150)x
T=7
B. y = 150x +950
C. x = 950y + 150
D. y = 950x + 150

Author: Victor A. Tondo Jr.


Author: Victor A. Tondo Jr.
29. Nelia takes ¾ hour to dress and get ready for 32. A man is 3 times as old as his son now. Four
school. It takes 4/5 hour to reach the school. If years ago, the sum of their ages was 36. Find the
her class starts promptly at 8:00 am; what is the man’s age now.
latest time she can jump out of bed in order not A. 33 B. 11
to be late for school? C. 29 D. 36
A. 6:42 am B. 6:27 am
C. 6:57 am D. 7:02 am
Solution:
First, create a table.
Solution:
Age Now Age 4 Yrs Ago
¾ hr = 45 mins, while 4/5 hr = 48 mins
45+48 = 93 mins, 93 mins = 1 hr 33 mins Man 3x 3x-4
Son x x-4
8:00 7:60
(3x-4) + (x-4) = 36
- 1:33  1:33
4x – 8 = 36
6:27
4x = 44; x = 11;
.: 1hr 33 mins before 8:00 AM is 6:27 AM 3x = 3(11) = 33

30. Which common fraction is equivalent to 33. What is the least common multiple of 12, 24
0.215? and 72?
A. 43/200 B. 27/125 A. 12 B. 72
C. 21/50 D. 108/375 C. 144 D. 36

Explanation: Explanation:
0.215 is read as 215 thousandths. In fraction Use continuous division.
form, that’s . In simplest form, .

Alternative Method: 34. The hypotenuse of a right triangle is 25 feet.


If one leg is 24 feet, what is the length of the
Just use your calculator. other leg?
A. 6 ft. B. 5 ft. C. 20 ft. D. 7 ft.
31. What are the next three terms in the
progression 1, 4, 16 …?
35. If two variables X and Y are directly related,
A. 64, 256, 1024 B. 67, 259, 1027 which of these is NOT true?
C. 48, 198, 1026 D. 65, 257, 1025
A. When X is low, Y is also low.
B. As X increases, Y also increases.
Explanation: C. When X increases, Y decreases.
D. A high Y is associated with a high X.
Each term is 4 times its precedent.

Solution:
C refers to an inverse or indirect relation.

Author: Victor A. Tondo Jr.


Author: Victor A. Tondo Jr.
36. Find the domain of f(x) = . Solution:
A. x B. x = 1 Since x – y = 3, then y – x = -3.
C. x = -1 D. x ,x -1 (-3)-3 = 1/(-3)3 = 1/-27 or -1/27

Explanation: 40. Factorize (x4 – 81) completely.


The given function is a rational algebraic A. (x-3)4
expression (RAE). When facing RAE, just look at B. (x – 3)2 (x + 3)2
the denominator and see if it can be equated to 0 C. (x+3) (x-3) (x2+9)
to make the RAE undefined. D. (x+3)3 (x-3)
The RAE will have an undefined value at
x = -1. Otherwise, it will always be equal to a real Solution:
number,
(x4 – 81) = (x2 – 9) (x2 + 9)
(x4 – 81) = (x+3) (x-3) (x2 + 9)
37. A car travels D km in H hours. Which of the
following expressions shows the distance
travelled by the car after M minutes? 41. √8 + √18 √2 = ____
A. MD/H B. 60MD/H A. 4√2 B. 5√2
C. MD/60H D. 60HD/M C. √24 D. 2√6

Solution: Solution:
Distance = Speed x Time (the unit of time should √8 + √18 √2 = 2√2 + 3√2 √2 4√2
be consistent)
The car is traveling at a speed of D/H km per hr.
The time is M minutes or M/60 hrs (for 42. By which property can we state the
consistency). following:
“If ax + b c, then ax + b - b = c – b.”
Distance = (D/H) (M/60) = MD/60H
A. transposition B. transitive
38. Find the surface area of a rectangular box C. additive inverse D. addition property
whose dimensions are 30 cm x 40 cm x 50 cm.
A. 4700 cm2 B. 7050 cm2
C. 9400 cm2 D. 11750 cm2 Explanation:
We added –b to both sides of the equation, thus
we used APE (addition property of equality).
Solution:
SA = 2 (LW + WH + LH)
SA = 2 (50x40 + 40x30 + 30x50) 43. The midpoint of P and (-7, 4) is (-3, 1). What
SA = 2 (2000 + 1200 + 1500) = 9400 are the coordinates of P?
A. (-5, 5/2) B. (-11, 7)
C. (1, -2) D. (-2, 3/2)
39. If x – y = 3, then (y-x)-3 = ___.
A. 9 B. -9
C. 1/27 D. -1/27

Author: Victor A. Tondo Jr.


Author: Victor A. Tondo Jr.
Solution: 47. How much water must be evaporated from
90 ml of a 50% salt solution to increase its
Let P be at (x,y). By Midpoint formula:
concentration to 75%?
(-7 + x)/2 = -3 (4 +y)/2 = 1
A. 40 ml B. 38 ml
-7 + x = -6 4+y=2
C. 35 ml D. 30 ml
x = -6 + 7 y=2–4
x=1 y = -2
Solution:
44. What is the slope of the line 3x – y = 11? V1 C1 + V2 C2 = VR CR
A. -1/3 B. 1/3 Since we are evaporating water, we will be
C. -3 D. 3 adding a NEGATIVE volume of water (or simply
put, we are subtracting water, diba?)
90(50) + (-X)(0) = (90-X)(75)
Solution:
4500 + 0 = 6750 – 75X
Isolate y on one side of the equation to rewrite 75X = 6750 – 4500
the equation in the form y = mx + b. 75X = 2250; X = 30
3x – y = 11
-y = -3x + 11
48. A and B form a vertical pair. If m A 3x
y = 3x – 11
and m B 5x – 44, what is the value of x?
A. 50.5 B. 28 C. 22 D. 16.75
45. What is the minimum value of
f(x) = 3x2 + 6x + 7?
Solution:
A. 1 B. -1
C. 4 D. -4 Since the two angles form a vertical pair, then
they are congruent.
3x = 5x – 44
Solution:
44 = 5x – 3x
Min Value = c – b2/4a 44 = 2x; 22 = x
That’s 7 – 36/12 or 7-3=4

49. The angle of elevation from an observer to


46. If xy = 23 and x2 + y2 = 75, find x + y. the top of a building is 30o. If the building is 50
A. 10.7845 B. 11 meters high, how far is the observer from the
C. 11.2155 D. 11.7845 building?
A. 25 B. 25√3
C. 50√3 D. 100
Solution:
x2 + 2xy +y2 = x2 + y2 + 2xy
x2 + 2xy +y2 = 75 + 2(23) Solution:
x2 + 2xy +y2 = 121 Use a 30-60-90 triangle. The side opposite of the
x + y = 11 30o angle will represent the building.

Author: Victor A. Tondo Jr.


Author: Victor A. Tondo Jr.
50. 1 and 3 are opposite angles in a Solution:
parallelogram. If m 1 40o, what is m 3? sin2 θ + cos2 θ 1
A. 40o B. 50o C. 70o D. 140o (0.28)2 + cos2 θ 1
cos2 θ 1 – 0.0784
cos2 θ 0.9216
Explanation: cos θ √0.9216 = ±0.96
Opposite angles of a parallelogram are
congruent.
54. If the sum of the supplement and the
complement of an angle is 130 degrees, what is
51. Two parallel lines are cut by a transversal, the angle?
forming H and K. If the two angles are A. 65o B. 70o
exterior angles on the same side of the C. 50o D. 25o
transversal, what is the measure of H if the
measure of K is 50o?
A. 25o B. 50o Solution:
C. 100o D. 130o (90-x) + (180-x) = 130
270 – 2x = 130
270 – 130 = 2x
Explanation: 140 = 2x
Exterior angles on the same side of the 70 = x
transversal are supplementary.
*Mnemonic: SST (same side of transversal) 55. If today is a Saturday, what day is 125 days
means supplementary. ALTERNATE (either from now?
interior or exterior) means congruent. Also,
CORRESPONDING angles are congruent. A. Thursday B. Friday
C. Sunday D. Monday

52. There are 33 red bags, 25 green bags, and 17


blue bags in a store. What percent of the bags is Solution:
red? Every 7 days, it would be a Saturday again.
A. 33% B. 44% The nearest multiple of 7 to 125 is 126. That
C. 66% D. 67% means 126 days after today is Saturday again,
and 125 days after today should be Friday.

Solution:
33/(33+25+17) = 33/75 or 11/25 56. Car A is traveling towards the east at a speed
11/25 in percent is 44% of 35 kph, while car B is traveling towards the
west at 45 kph. If they left the same point at 1:00
PM, how far apart are they at 3:45 PM?
53. Given sin θ 0.28, which of the following A. 240 km B. 220 km
could possibly be cos θ? C. 200 km D. 180 km
A. 0.72 B. -0.86
C. 0.96 D. 1.14

Author: Victor A. Tondo Jr.


Author: Victor A. Tondo Jr.
Solution: Solution:
Time spent driving: 1:00 to 3:45 = 2.75 hrs Let x = lowest even number
(45 mins in decimals is 45/60 since there are 60 x + (x+2) + (x+4) + (x+6) + (x+8) = 120
mins in 1 hr) 5x + 20 = 120
Car A distance from mid: 2.75 (35) = 96.25 5x = 100
Car B distance from mid: 2.75 (45) = 123.75 x = 20; .: numbers are 20, 22, 24, 26, 28
Total distance: 123.75 + 96.25 = 220 km 22 + 28 = 50

Alternative Solution: Alternative Solution:


Since the two cars are traveling in two opposite The middle (3rd) even number is 120/5 or 24.
directions, add their speeds and multiply by That means the 2nd even number is 24-2 or 22,
elapsed time. and the 5th is 24+2(2) or 28.
2.75 (45+35) = 2.75 (80) = 220 km
59. If x = 3, which of the following is equal to 13?
57. Mr. Santos left the house at 1:00 PM and A. 5x + 2 B. x2 + 2x + 1
traveled east at an average speed of 40 kph. His C. x3 – 4x – 2 D. x2 + x + 2
wife Mrs. Santos left the at 2:00 PM and traveled
west at an average speed of 30 kph. How far
apart are they at 4:00 PM? Explanation:
A. 180 km B. 140 km Just substitute x with 3.
C. 100 km D. 60 km

60. If f(x) = x2 + 4x + 3, which of the following is


Solution: equal to 99?
Mr. Santos’s data: A. f(11) B. f(-12)
Speed: 40 kph C. f(12) D. f(-8)
Elapsed time: 1PM to 4PM = 3 hrs
Distance: 40kph (3hrs) = 120 km Solution:
x2 + 4x + 3 = 99
Mrs. Santos’s data: x2 + 4x + 3 + 1 = 99 + 1
Speed: 30 kph x2 + 4x + 4 = 100
Elapsed time: 2PM to 4PM = 2 hrs
Distance: 30 kph (2hrs) = 60 km √ +2 100
x + 2 = ± 10
Total Distance: 60 + 120 = 180 km x = -2 ± 10 That’s -2+10 or 8, and -2-10 or -12

58. Five consecutive even numbers have a sum of 61. Given f(x) = ln , what is f ‘ x ?
120. What is the sum of the 2nd and 5th even
numbers? A. B.
A. 46 B. 48 C. (2x+2) ln (x2+2x) D. 2x + 2
C. 50 D. 52

Author: Victor A. Tondo Jr.


Author: Victor A. Tondo Jr.
Solution: Explanation:
You can rewrite ln as x2 + 2x since ln is When x is the squared variable, the parabola
the natural logarithm (the logarithm whose base opens upward when the coefficient of y is
is the natural number, e). positive (example: A).
Remember: ln eu = u, wherein u is the exponent When x is the squared variable, the parabola
to which e is being raised. opens downward when the coefficient of y is
negative (example: B).
The derivative of x2 + 2x is, of course, 2x + 2.
When y is the squared variable, the parabola
opens to the left when the coefficient of x is
62. Which of the following could be the value of x negative (example: C).
if x 3(mod 11)? When y is the squared variable, the parabola
A. 33 B. 47 C. 52 D. 2 opens to the right when the coefficient of x is
positive (example: D).

Solution:
66. Factorize: 12x2 – 7x – 10.
Just divide the numbers by 11 and see which one
gives a remainder of 3. A. (6x + 5) (2x – 2) B. (6x – 2) (2x + 5)
C. (3x + 2) (4x – 5) D. (3x – 2) (4x + 5)

63. If = 6x2 + 8x – 7, which could be u?


67. For which value of k does 4x2 + kx + 49 have
A. 12x + 8 B. 3x3
+ 4x2
– 7x + 11 only one root?
C. 2x3 + 4x2 -7x +1 D. 12x2 + 8x - 10
A. -28 B. -14 C. 7/2 D. -7/4

Explanation:
Explanation:
Anti-derivatives. If you already forgot how to do
You may use Completing Square Trinomials. The
that, simply check which choice has a derivative
middle term is twice the product of the square
of 6x2 + 8x – 7.
roots of the first and third terms. In the problem,
the middle term is twice the product of √4 and
64. What is the center of x2 + y2 – 8x + 6y = 0? √49. That’s 2 2x 7 or 28x. Don’t forget that the
middle term could be positive or negative.
A. (-8.6) B. (8, -6)
C. (-4, 3) D. (4, -3)
You may also use the discriminant to answer
this:
Solution:
b2 – 4ac 0 when there’s only one root,
The center, C(h,k) is given as h = -D/2 and
b2 – 4ac > 0 when there are two real roots
k = -E/2 wherein D and E are from the equation
b2 – 4ac < 0 when there are no real roots
x2 + y2 + Dx + Ey + F = 0.

68. If A and B are the roots of x2 + 7x + 15, what


65. Which of the following is a parabola that
is AB?
opens to the right?
A. 6y = (x+9)2 - 8 B. -4y = (x-6)2 + 3 A. 7√3 + 2 B. 2√3 + 7
C. -5x + 3 = (y-2)2 D. 2x + 6 = (y+3)2 C. 3√2 + 2√3 D. 15
Author: Victor A. Tondo Jr.
Author: Victor A. Tondo Jr.
Explanation: 71. How many terms are there in the sequence
5, 13, 21, 29, …, 357?
Since A and B are the roots, then AB pertains to
the product of the roots which is given as c/a. A. 40 B. 44
C. 45 D. 70

69. 1 + 2 + 4 + 8 + … + 2048 ____


Solution:
A. 4095 B. 4096
C. 4097 D. 4098 An = A1 + (n-1)d
357 = 5 + (n-1)(8)
357-5 = 8(n-1)
Solution: 352 =8(n-1)
You may use the Geometric Series formula which 44 = n-1
45 = n
is ∑ ( ), where r is the common
ratio, n is the number of terms, and a1 is the first
term. Alternative Solution:
(this is the “y=mx+b” solution I taught my grade
Alternative Solution: 3 student for Singapore. Yes, Grade 3.)

In this problem, however, you cannot easily use Before anything else, since this might be “new”
the GS formula since you don’t know n, the to you, your d is our m, your An is our y, your n is
number of terms. our x, and b is your A1 – d.

I will personally use the shortcut for the sum of a 357 = 8x +(5-8)
geometric sequence wherein the ratio is 2 or ½. 357 = 8x – 3
The shortcut is SUM = 2(largest) – smallest. In 360 = 8x
this problem, that’s 2 2048 -1 = 4095. 45 = x  It’s a lot shorter and a lot faster
if you’re used in Tagalog, “sanay” to it.
You may also apply this in the next item, #70.

72. How many ways can a group of 5 be selected


70. 24 + 12 + 6 + 3 + 1.5 + … ____ from 5 boys and 5 girls if the group must contain
A. 48 B. 50 C. 54 D. 60 3 boys and 2 girls?
A. 151,200 B. 1200
C. 252 D. 100
Solution:
You may use the Infinite Geometric Series
Solution:
formula which is ∑ ( ), where r is the
common ratio and a1 is the first term. A group, committee, or team (any set with no
hierarchy of members) calls for Combinations.
To pick 3 boys from a total of 5 boys, use 5C3
Alternative Solution: and that’s 10. To pick 2 girls from a total of 5
In this problem, I would still be using the girls, use 5C2 and that’s 10. Lastly, 10x10 100.
shortcut since the ratio is ½. Since this is an
infinite geometric sequence, then the last term
won’t have any significant value. Thus, the sum is
simply twice the first term. That’s 2 24 48.

Author: Victor A. Tondo Jr.


Author: Victor A. Tondo Jr.
73. What is the probability of getting a sum of 9 Solution:
when rolling 2 dice? Simply interchange the numerical coefficients of
A. 1/9 B. 5/36 x and y in the original equation, then change the
C. 1/6 D. 7/36 operation between them. 2x – 3y becomes
3x + 2y. For the constant, simply substitute the
x and y values of the point ((1,2) in this
The only pairs with a sum of 9 are (3,6), (4,5), problem) and solve for the constant.
(5,4), and (6,3). There are only 4 pairs out of 36. 3(1)+2(2)=7. So we have 3x+2y=7.

̅̅̅̅ where A is at (-3,4)


74. C is the midpoint of AB 77. Two parallel lines are cut by a transversal to
and B is at (7,-10). Find the coordinates of C. form X, Y, and Z. Given that X and Y are
A. (5,-7) B. (-5,7) C. (2,-3) D. (-2,3) alternate interior angles while Y and Z are
interior angles on the same side of the
transversal, find m Z if m X 40o.
Solution: A. 40o B. 50o
Midpoint Formula: ( , ) C. 130o D. 140o

Midpoint: ( , )
Explanation:
Alternate, corresponding, and vertical pairs
75. It is a line segment formed by connecting two automatically suggest that the two angles are
non-consecutive vertices of a polygon. congruent. Linear pairs and angles on the same
side of transversal (SST) are supplementary.
A. side B. apothem
C. altitude D. diagonal m X 40, .: m Y 40 since alternate interior
angles
m Z 180-40 140 since Y and Z are
Explanation: interior angles on the same side of the
A side is formed by connecting two consecutive transversal.
vertices of a polygon.
The apothem is only for regular polygons. It is 78. The measure of each interior angle of a
the perpendicular bisector of one of its sides, regular polygon is 144o. How many vertices does
passing through the center. it have?
A diagonal is a line segment formed by A. 36 B. 24 C. 12 D. 10
connecting two non-consecutive vertices of a
polygon.
Solution:
76. Find the equation of the line perpendicular to MIA = =
2x – 3y = 7, passing through (1,2).
Alternative Solution:
A. 2x + 3y = 8 B. 3x + 2y = 7
C. 2x – 3y = -4 D. 3x – 2y = -1 Personally, I always go for the exterior angle first
to get the number of sides or vertices. Since the
exterior and interior are supplementary, then
each exterior measures 180-144 or 36. The
formula for number of sides or vertices given the
Author: Victor A. Tondo Jr.
Author: Victor A. Tondo Jr.
measure of each exterior is 360÷MEA, so that’s Solution:
360÷36 or 10 vertices. x(x+2) = 3248
By the way, you may derive this solution by x2 + 2x = 3248
manipulating the formula for MIA: . That x2 + 2x + 1 = 3249
becomes 180 – . That means = 180 – MIA, √ +2 +1 √3249
x + 1 = 43
or = n.
x = 42

79. Solve: (x + 9) (x – 3) < 0 WAIS Solution:


A. -9 < x < 3 B. x < -3 x > 9
Get your scientific calculator, extract √3248 and
C. x < -9 x > 3 D. x ; x -9, 3
then scrape the decimals or round down.
#2EZ4U
Solution:
Usually, people would 81. Solve for x: 2log2 3 – log2 18 = x
straight go for the Test
A. ½ B. -1 C. -2 D. 1
Point Table method which
we use in Calculus.
However, since this is the Solution:
licensure exam, I’d prefer
that you use a simpler and Rewrite the logarithm as a single logarithm by
quicker approach to this applying the rules of logarithms.
problem.
2log2 3 becomes log2 , or log2 ½
First, identify the zeros of log2 ½ = -1
the inequality by equating
each factor to 0. Our zeros
are -9 and 3. 82. Twinkle Bucks has four serving sizes for their
Next, identify the opening milk tea: Small, Medium, Large, and Extra Large.
of the parabola. Since the What level of data are they using for their
leading coefficient would serving sizes?
be positive, then the parabola opens upwards. A. nominal B. ordinal
Now, since the parabola opens upwards, then the C. interval D. ratio
parts less than 0 should be between the zeros of
the inequality. That means x should be between -
83. After receiving a 20% markup, a bag was sold
9 and 3.
for P960. How much was it originally?
A. P1152 B. P4800
80. The product of two consecutive even C. P800 D. P1200
counting numbers is 3248. Find the smaller
number.
A. 42 B. 46 Solution:
C. 52 D. 56 Selling Price = Original Price (1 + Markup Rate)
960 = OP (1 + 0.20)
960/1.2 = OP
800 = OP
Author: Victor A. Tondo Jr.
Author: Victor A. Tondo Jr.
84. Given ̅̅̅̅
BT bisects ABC and m ABT = 40o, 87. A 10 ft ladder leans against a wall, forming a
find m ABC. 30o angle with it. How high on the wall does it
reach?
A. 20o B. 40o C. 60o D. 80o
A. 5 ft B. 5 √3 ft
C. 10 √3 ft D. 10 √6 ft
Explanation:
ABT is formed after the bisection of ABC. That
means ABT is half of ABC, or ABC is twice of Solution:
ABT. Draw the problem first. The ladder and the wall
form a 30o angle with each other and the wall is
of course perpendicular to the ground.
85. A cone has a radius of 9 cm and a slant height
of 15 cm. Find its volume. That means the ladder forms a 60o angle with the
ground. The ladder is the hypotenuse, while its
A. 243 π cm3 B. 324 π cm3 reach on the wall is adjacent to the 30o angle or
C. 405 π cm3 D. 486 π cm3 simply put, the longer side. The smaller side
measures half of 10 or 5 ft, therefore the longer
Solution: side must be 5√3 ft.

Be careful with cones. Tendency kasi sa LET that


they will give the slant height while looking for 88. How many ways can a committee of 5 be
volume and the height while looking for the selected from 9 people?
surface area. Just remember that the slant height
A. 126 B. 120
is always longer than the height. The slant height
C. 3024 D. 15120
is the hypotenuse, while the height is one of the
legs with the radius as the other. Just use the
Pythagorean formula to solve for whichever is Explanation:
missing.
Just use your scientific calculator: 9C5.
The height is 12 cm (after using Pythagorean
formula).
Vol = π r2 h = π 92 12 324 π cm2 89. What is 60% of 80% of 500?
A. 480 B. 240
C. 120 D. 60

86. If f(x) = x2 + 4x + 4 and g(x) = x-2, find Solution:


f(g(x)).
(0.6)(0.8)(500) = 240
A. x2 B. x3 – 6x2 + 6x – 9
C. x2 + 8x + 16 D. x2 – 8x + 16
90. If 3x = 7 and 2y = 5, what is 6(x-y)?
Solution: A. -1 B. 1-√35
f(g(x)) = f(x-2) C. √7 - √5 D.
= (x-2)2 + 4(x-2) + 4
= (x2 – 4x + 4)+ (4x – 8) + 4 = x2
Solution:

Author: Victor A. Tondo Jr.


Author: Victor A. Tondo Jr.
6(x-y) = 6x – 6y = 2(3x) – 3(2y) For the last digit, since we have already used two
2(3x) – 3(2y) = 2(7) – 3(5) = 14-15 = -1 digits, we only have 4 choices.
5 x 5 x 4 = 100
91. If two numbers have a product of 71 and the
sum of their squares is 147, what is their sum? 94. How many ml of 20% acid must be added to
A. -17 B. 5 400 ml of 50% acid to make a 30% acid solution?
C. 12√3 + √5 D. 12 + √3 A. 1000 ml B. 900 ml
C. 800 ml D. 750 ml
Solution:
Let A and B be our two numbers. Solution:
AB = 71; A2 + B2 = 147 C1 V1 + C2 V2 = CR VR
.: A + B + 2AB = 147 + 2(71) = 289
2 2
20 (V) + 50 (400) = 30 (V + 400)
(A + B)2 = 289; A + B = ±17 20V + 20,000 = 30V + 12,000
20,000 – 12,000 = 30V – 20 V
8,000 = 10V; 800 = V
92. Find the median: 7, 9, 11, 10, 9, 13, 17, 14
A. 10 and 11 B. 9 and 10
C. 10.5 D. 9.5 95. How many ml each of 10% and 50% solution
should be mixed to make 500 ml of 18%
solution?
Solution:
A. 400 ml of 10% and 100 ml of 50%
Rearrange the numbers from least to greatest: B. 350 ml of 10% and 150 ml of 50%
7, 9, 9, 10, 11, 13, 14, 17  there are 8 nos. C. 300 ml of 10% and 200 ml of 50%
D. 200 ml of 10% and 300 ml of 50%
The median is the th or 4.5th number. That
means we have to get half the sum of our 4th and
5th numbers. (10+11)/2 = 10.5 Solution:
Since our resultant volume is 500, then our two
volumes will be x and (500-x).
93. How many 3-digit numbers can be formed
using the digits 0, 1, 2, 3, 4 and 5 if repetition is C1 V1 + C2 V2 = CR VR
not allowed? 10(x) + 50(500-x) = 18(500)
A. 60 B. 80 C. 100 D. 120 10x + 25,000 – 50x = 9,000
25,000 – 9,000 = 50x – 10x
Solution: 16,000 = 40x; 400 = x
Use FCP (Fundamental Counting Principle): 96. It takes 28 men a total of 24 days to build a
__ x __ x __ house. How long would it take 32 men to build a
similar house?
For the first digit, we cannot use 0. That means
we only have 5 choices for the first digit. A. 28 days B. 27 days
For the second digit, we can now use 0. Since we C. 21 days D. 19 days
have already used one digit for the first, that
means we have 5 choices for the second digit.

Author: Victor A. Tondo Jr.


Author: Victor A. Tondo Jr.
Solution: An isosceles trapezoid has congruent diagonals,
however, they do not bisect each other, nor are
This is an indirect or inverse proportion.
they perpendicular.
Let x = number of days it would take the 32 men
A rhombus has diagonals that are perpendicular
to build the house
and that bisect each other. However, they are not
28(24) = 32 x congruent.
672 = 32 x
PS: A square has diagonals that are congruent,
21 = x
perpendicular, and that bisect each other.

97. Evaluate: lim 100. A pipe can fill a pool in 6 hours while
another pipe can drain empty the pool in 15
A. undefined B. limit does not exist
hours. How long will it take to fill the pool if both
C. 8 D. +
pipes are open?
A. 9 hours B. 9.125 hours
Explanation: C. 9.45 hours D. 10 hours
You may simplify the function first before
substituting x with 4.
Solution:
This is similar to our “Working Together”
98. A box contains 7 red, 8 blue, and 9 white problem, except instead of adding their times, we
balls. When taking two balls in succession, what will subtract (since the draining pipe is doing the
is the probability that both balls are white? opposite of helping).
A. 9/64 B. 9/69 AB/(A-B) = 15(6)/(15-6) = 90/9 = 10 hrs
C. 7/64 D. 7/69
101. If log n – 1 = 2, find n.
Solution: A. 3 B. 1000 C. e3 D. 3e
First white ball: 9/24
Second white ball: 8/23
Solution:
9/24 x 8/23 = 9/69
log n – 1 = 2
log n = 2 + 1 = 3
99. Which of the following has two diagonals that
(note that the base of the log is 10)
are perpendicular bisectors of each other?
log n = 3 translates to 103 = n
A. kite B. rectangle Therefore n = 1000
C. rhombus D. isosceles trapezoid

102. log2 3 + 2 log2 7 – log2 5 = ______.


Explanation:
A. log2 B. log2
A kite’s diagonals are perpendicular but only one
diagonal will bisect the other. C. log2 D. log2
A rectangle’s diagonals are congruent and they
bisect each other. However, they are not
Explanation:
perpendicular.
Just apply the laws of logarithms.
Author: Victor A. Tondo Jr.
Author: Victor A. Tondo Jr.
103. The surface areas of two spheres are 12 π If x is even, x = 2y where y is any counting
cm2 and 108 π cm2. What is the ratio of their number. 4(4y2) + 4(2y) = 16y2 + 8y, which is
volumes? also divisible by 8.
A. 1:3√3 B. 1:9 Either ways, the statement holds true.
C. 1:27 D. 2:3√3
106. Find the volume of a steel cylinder of radius
Solution: 5 cm and height 12 cm.
Ratio of surface areas: 12:108 or 1:9 A. 300 π cm3 B. 250 π cm3
Ratio of radii: √1: √9 or 1:3 C. 200 π cm3 D. 100 π cm3
Ratio of volumes: 13:33 or 1:27
Solution:
104. The volume of a regular hexahedron is 64 Vol π r2 h = 52 12 π 300 π cm3
in3. How long is each side?
A. 2 in B. 4 in C. 6 in D. 8 in
107. A cube sits perfectly inside a sphere of
volume 108 √3 π cm3. Find the volume of the
Explanation: cube.
A regular hexahedron is simply a cube. A. 27 cm3 B. 54 cm3
105. Which of the following statements is C. 108 cm3 D. 216 cm3
ALWAYS true? Solution:
A. The square of a prime number is odd. Volume of sphere = 108√3 π cm3
B. The sum of two consecutive even numbers is
π r3 = 108√3 π
divisible by 4.
C. Any even number is composite. r3 = (108)√3
D. The product of two consecutive even numbers r3 = 81√3; r = 3√3; d = 6√3
is divisible by 8.
Diagonal of cube = s√3 = 6√3
.: s = 6; volume = s3 = 63 = 216
Explanation:
A. Counterexample: The prime number, 2. The
square of 2 is 4 which is even. Alternative Solution:
B. Always false. One example is 2 and 4. Their Ratio of volume of cube to sphere (cube is inside
sum, 6, is not divisible by 4. sphere) = 2 : √3 π
C. Counterexample: The prime even number, 2. N : 108 √3 π 2 : √3 π
( √ )
D. Proof by Algebra: Let the first even number be N= 216

2x. The second even number will be 2x + 2.
Their product will be 4x2 + 4x.
108. Find the distance in cm of an 80 cm chord
If x is an odd number, x = 2y + 1 where y is a
from the center of a circle whose radius is 41 cm.
counting number. 4x2 + 4x = 4(4y2 + 4y + 1) +
4(2y + 1) = 16y2 + 16y + 4 + 8y + 4 = 16y2 + A. 41 - 2√10 B. 41 - 4√10
24y + 8, which is divisible by 8. C. 9√2 D. 9

Author: Victor A. Tondo Jr.


Author: Victor A. Tondo Jr.
Solution: Solution:
The chord is perpendicularly bisected by a y = x2 + 10x
segment connected to the center of the circle, y + 25 = x2 + 10x + 25
whose length is the distance we are looking for. y + 25 = (x+5)2
If the radius is drawn connected to one endpoint √ + 25 = x + 5
of the chord, we can form a right triangle whose √ + 25 – 5 = x
hypotenuse is the radius and one leg is half of the
chord. Using the Pythagorean theorem, the √ + 25 – 5 = y-1
distance is √41 40 or simply, 9.
113. Find the intersection of y = 2x + 3 and
y = 4x – 11.
109. Which quadrilateral has two congruent
diagonals that bisect each other? A. (-4/3, 0) B. (4/3, 0)
C. (7, 17) D. (-7,-17)
A. kite B. isosceles trapezoid
C. rectangle D. rhombus
Solution:
110. What is the longest side of ∆MTC if m M = y = 2x + 3
40o and m C = 60o? - y = 4x – 11
̅̅̅̅
A. MC ̅̅̅̅
B. TC ̅̅̅̅
C. MT ̅̅̅̅
D. CT 0 = -2x + 14
2x = 14; x=7
114. Find the area of the triangle whose vertices
Explanation: are (1,4), (2,3), and (3,0).
m T = 180-(40+60) = 80 A. 0 B. 1 C. 5/3 D. 3/4
The longest side is opposite the largest angle, T.

Solution:
111. Find the altitude to the hypotenuse of a
right triangle whose legs measure 10 cm and 24 1 2 3 1
| | | = 2 1
cm. 4 3 0 4
Note: If the result is negative, that means your
A. 120 cm B. cm points are simply arranged clockwise. Just get
C. 120√2 cm D. 24√5 cm the absolute value of the answer.
115. Find the tenth term: 3, 10, 17, 24, …
Solution: A. 66 B. 67 C. 68 D. 69
Find the hypotenuse first. That would be 26 cm.
Altitude to the Hyp = (L1 L2)/Hyp = 24(10)/26 Solution:
Altitude to the Hyp = 120/13
A10 = 3 + (10-1) (7) = 3 + 63 = 66

112. Find the inverse of y = x2 + 10x.


116. Find the remainder when
A. y-1 = √ 25 + 5 x4 – 5x3 + 6x2 + 2x + 1 is divided by (x – 2).
B. y-1 = √ 25 – 5
A. 17 B. 13 C. 9 D. 5
C. y = √ + 25 + 5
-1

D. y-1 = √ + 25 – 5

Author: Victor A. Tondo Jr.


Author: Victor A. Tondo Jr.
Solution: 120. If 2x = 3y and 4y = 5z, what is z in terms of
x?
24 – 5(23) + 6(22) + 2(2) + 1
= 16 – 40 + 24 + 4 + 1 = 5 A. z = x B. z = x
C. z = x D. z = x
117. The sum of Fe’s age and Sita’s age is 60.
Twelve years ago, Fe was twice as old as Sita.
How old is Sita now? Solution:
A. 18 B. 24 C. 30 D. 36 Make two equations wherein y will have the
same numerical coefficients.
2x = 3y  8x = 12y
Solution:
4y = 5z  12y = 15z
Age Now Age 12 yrs ago
By transitive property of equality,
Fe x x – 12
Sita 60-x (60-x) – 12 or 48-x 8x = 15z, or x = z

x-12 = (2)(48-x)
x-12 = 96 – 2x 121. Victor had an average of 94 on his first four
3x = 96 + 12 = 108 Math tests. After taking the next test, his average
x = 36; 60-x = 60-36 = 24 dropped to 93. Find his most recent grade.
118. If the length of a rectangle is increased by A. 88 B. 89 C. 90 D. 91
20% while the width is decreased by 10%, what
will happen to its area?
Solution:
A. decreased by 10%
B. increased by 10% New Score =(New Number)(New Average) –
C. increased by 8% (Old Number)(Old Average)
D. decreased by 2% New Score = 5(93) – 4(94) = 465 – 376 = 89

Solution: 122. X is of Y and Y is of Z. What part of Z is X?


(L x 1.2) (W x 0.9) = (1.08 x LW)
A. X = Z B. X = Z
C. X = Z D. X = Z
119. The 19th term of an arithmetic sequence is
85 and the 12th term is 43. Find the common
difference.
Solution:
A. 5 B. 6 C. 7 D. 8
X= Y

Solution: X = ( Z) = Z or Z

d= = 6

Author: Victor A. Tondo Jr.


Author: Victor A. Tondo Jr.
123. Two buses leave the same station at 8:00 Solution:
pm. One bus travels north at the rate of 30 kph To rationalize this, multiply both numerator and
and the other travels east at 40 kph. How many denominator by the conjugate of the
kilometers apart are the buses at 10 pm? denominator. By doing this, we are sure to have
A. 140 km B. 100 km a rational denominator.
C. 70 km D. 50 km √ √ √
x
√ √ √
Solution:
From 8 to 10 PM is 2 hours. After two hours, one 127. RNHS has 130 quizzers. 67 of them are
bus will have travelled 60 km while the other, 80 Math, 60 are Science, and 20 are quizzers for
km. Since the two buses are traveling on both Math and Science. How many quizzers are
perpendicular directions, we can use the neither Math nor Science?
Pythagorean Theorem to find their distance.
A. 0 B. 13 C. 17 D. 23
D = √60 + 80 = 100 km

Solution:
124. A bus drove for 6 hours at 75 kph and 4
(A B ’ U - (A B)
hours at 80 kph. What was its average speed?
(A B ’ 130 – [A + B – (A B)]
A. 76 kph B. 77 kph (A B ’ 130 – (67 + 60 – 20) = 130 – 107 = 23
C. 77.5 kph D. 78 kph

128. Mr. Tondo has P100,000 to invest, from


Solution: which he wants to earn P5600 per year. Bank A
Get the total distance and the total time first. offers 5% per annum while Bank B offers 6%.
How much should he invest at Bank B?
6 hrs x 75 kph = 450 km
4 hrs x 80 kph = 320 km A. P45,000 B. P50,000
C. P55,000 D. P60,000
Total distance = 770 km, total time = 10 hrs
Average spd = = = 77 kph
Solution:
Let x = investment in Bank B
125. 18 students failed a quiz. They represent .: 100,000 – x = investment in Bank A
30% of the class. How many students passed the 0.05(100,000 – x) + 0.06x = 5,600
quiz? (5,000 – 0.05x) + 0.06x = 5,600
A. 60 B. 42 C. 36 D. 24 0.01x = 600; x = 60,000

Solution: 129. Evaluate when x = ¾ and y = .


18:30% = N:70% A. -38 B. -19 C. 19 D. 38
18(70)/30 = N; 42 = N
126. Rationalize:


A. +1 B. 2√5 – 4
√ √
C. D.
Author: Victor A. Tondo Jr.
Author: Victor A. Tondo Jr.
Solution: 133. How many line segments can be made from
30 non-collinear points?
+ ;
A. 900 B. 870 C. 450 D. 435
÷ x 19

Solution:
130. Today, Vic is 11 years old while his father is 30C2 = 435
37. How many years from now will his father be
twice as old as he?
134. The longest chord of a circle is 80 cm. How
A. 15 B. 13 C. 11 D. 10
long is its radius?
A. 20 cm B. 30 cm
Solution: C. 20√2 cm D. 40 cm
Let x = number of years from now
2(11+x) = 37+x Explanation:
22 + 2x = 37 + x
2x – x = 37 – 22; 15 = x The longest chord is the diameter, and the radius
is half the diameter.

131. Carla and Diana are on a seesaw. Carla


weighs 50 kg and sits 168 cm to the left of the 135. Find k such that 34k67 is divisible by 9.
fulcrum. If Diana weighs 60 kg, how far to the A. 5 B. 6 C. 7 D. 8
right of the fulcrum must she sit to balance the
seesaw?
Solution:
A. 140 cm B. 170.8 cm
Remember that for a number to be divisible by 9,
C. 201.6 cm D. 210 cm
the sum of its digits must be equal to 9.
3+4+k+6+7 = 20+k
Solution: 2+0+k = 2 + k = 9; k=7
Seesaw problems call for inverse or indirect
proportion.
136. Find the largest area of a rectangle whose
50(168) = 60N perimeter is 100 cm.
8400 = 60N
A. 2500 cm2 B. 2499 cm2
140 = N
C. 625 cm2 D. 624 cm2

132. Twenty guests shake hands with each other.


Solution:
If each guest is to shake hands with all the other
guests, how many handshakes will be made? Instead of jumping to differential calculus
(minima and maxima) to solve this, simply make
A. 400 B. 380 C. 200 D. 190
it a square. That’s the shortcut for this kind of
Solution: question.
20C2 = 190 137. What time is 200 minutes past 10:30 PM?
A. 12:30 AM B. 12:30 PM
C. 1:50 AM D. 1:50 PM
Author: Victor A. Tondo Jr.
Author: Victor A. Tondo Jr.
Solution: 141. If x is 80% of y, what percent of y is x?
200 minutes = 3 hrs 20 mins A. 120% B. 125%
C. 130% D. 135%
10:30 PM
+ 3:20
13:50 PM  1:50 AM Solution:
x = 0.8 y; y 1÷0.8 = 1.25
.
138. Find the product of two numbers whose 1.25x = y
GCF is 24 and LCM is 120.
A. 2880 B. 1440 C. 720 D. 360
142. Bus X left the terminal at 1 PM and traveled
at a speed of 60 kph. Bus Y left the same terminal
Explanation: 2 hours later and traveled 80 kph on the same
route. What time will Bus B catch up with Bus A?
The product of two numbers is equal to the
product of their LCM and GCF. A. 6 PM B. 9 PM
C. 11 PM D. 1 AM
24 x 120 = 2880

Solution:
139. The salary of 4 men for 5 days is P9,000.
How much is the salary of 5 men for 6 days? Let N = running time for Bus X
A. P12,000 B. P12,600 60x = 80(x-2)  Bus Y left 2 hrs later
C. P13,500 D. P14,400 60x = 80x – 160
160 = 20x; 8 = x
8 hours after Bus Y left the terminal is 11AM.
Solution:
First, find the cost of each “man-day”.
143. What is the degree of the polynomial
4 men x 5 days = 20 man-days
-3 x2y3 + 21 x3y4 – 7 x5y6 – 15?
P9,000 ÷ 20 man-days = P450 per man-day
A. 4 B. 5 C. 11 D. 21
You may now solve the problem.
5 men x 6 days = 30 man-days
30 man-days x P450 per man-day = P13,500 Explanation:
The degree of a polynomial is the highest sum of
exponents in a term.
140. The average grade of eleven students is 83.
If the average of six of these students is 88, what
is the average of the other 5 students? 144. The average of x+5, 2x-4, and x+7 is 20.
A. 77 B. 78 C. 79 D. 80 Find x.
A. 18 B. 13 C. 9 D. 8

Solution:
Sum of grades of 11 students: 11x83 = 913 Solution:
Sum of grades of 6 students: 6x88 = 528 20
Sum of grades of other five: 913 – 528 = 385 (x+5) + (2x-4) + (x+7) = 60
Average of grades of other five: 385÷5 = 77 4x + 8 = 60; 4x = 52; x = 13
Author: Victor A. Tondo Jr.
Author: Victor A. Tondo Jr.
145. Mia is 16 years younger than Kia. 13 years 148. Adam can do a job alone in 8 hours, while
ago, Kia was thrice as old as Mia. What is Kia’s Bam can do the same job in 12 hours. One day,
present age? they worked together for 1 hour before Bam left
Adam to finish the job. How long will it take
A. 43 B. 40 C. 37 D. 34
Adam to finish the remaining job?
A. 6 hrs 50 mins B. 6 hrs 40 mins
Solution: C. 6 hrs 30 mins D. 6 hrs 20 mins
Age today Age 13 years ago
Kia x x – 13
Solution:
Mia x – 16 x – 16 – 13 or x – 29

(x – 13) = 3(x – 29) ,

x – 13 = 3x – 87 = or 6 hrs
-13 + 87= 3x - x
74 = 2x; 37 = x That’s 6 hrs and 20 mins

146. Insert one term between 18 and 32 to make Mnemonic:


a geometric sequence.
For questions like this (about working together
A. 20 B. 24 C. 25 D. 27 and then someone leaves), use PuTS U. When
someone leaves you, “PuTS U” !!

Solution: PuTS U stands for P(u)roduct, Time, Sum, Umalis

Shortcut for inserting one term is √AB. This is


also the formula for the geometric mean. 149. Find x if 2748 = 9x.
√18 32 √576 24 A. 144 B. 81
C. 72 D. 60

147. There are 100 pigs and chickens in a farm,


all of which are healthy. If there are 340 legs in Solution:
total, how many pigs are there?
Express both sides as a power of 3.
A. 70 B. 65
(33)48 = (32)x
C. 60 D. 55
3144 = 32x
Solution: 144 = 2x; 72 = x
Let P = number of pigs; 150. Solve for x: 49x = 343
C = number of chickens
A. 1.142857 B. 7
P + C = 100 C. 1.5 D. √7
4P + 2C = 340  since pigs have four legs
and chickens have two
2(P + C = 100)  2P + 2C = 200 Solution:
4P + 2C = 340 - 4P + 2C = 340 First, express both numbers as powers of the
-2P = - 140 same base.
P = 70 49x = 343  (72)x = 73

Author: Victor A. Tondo Jr.


Author: Victor A. Tondo Jr.
Apply the laws of exponents 2 +1 4
(72)x = 73  72x = 73 154. Given { 4 4,
2x = 3; x = 3/2 or 1.5 7 4
find lim
A. 4 B. 9
151. What is the highest possible product of two
C. 0 D limit does not exist
numbers if their sum is 45?
A. 506 B. 506.25
C. 506.5 D. 506.725 Solution:
Limit from the left: 2(4) + 1 = 9
Limit from the right: 42 – 7 = 9
Solution:
Since both limits are equal, then the limit is 9.
Instead of jumping straight to minima and
maxima under differential calculus, simply make
your numbers equal to maximize their product. 155. If today is a Saturday, what day is 125 days
from now?
45/2 = 22.5
A. Friday B. Sunday
Both numbers will be 22.5, so their product is
C. Monday D. Tuesday
22.5 x 22.5 = 506.25

Solution:
152. Which statistical test is used for comparing
observed frequencies to expected frequencies? This is an application of modulo.
A. ANOVA B. t-test 125 6 (mod 7) or 125÷7 = 17 r. 6
C. Pearson R D. Chi Square 6 days after Saturday is Friday

Explanation: 156. If the sum of the supplement and the


complement of an angle is 124, what is the
Observed vs Expected: Chis Square
angle?
Relationship: Pearson R (R for relationship)
Group differences: ANOVA (variance = A. 71 B. 72 C. 73 D. 74
differences)
Comparing sets of normal distributions: T-test
Solution:
(180-x) + (90-x) = 124
153. The product of two consecutive odd 270 – 2x = 124
counting numbers is 1443. What is their sum? 270 – 124 = 2x
A. 76 B. 78 C. 80 D. 82 146 = 2x; 73 = x

Solution: 157. Find + given x + y = 20 and xy = 81.


Let x = first number; x+2 = next number
x(x+2) = 1443 A. B. C. D.
x2 + 2x = 1443
x2 + 2x + 1 = 1443 + 1
√ + 2x + 1 = √1444 Solution:
x+1 = 38; x = 37 x+2 = 39
+ +
Author: Victor A. Tondo Jr.
Author: Victor A. Tondo Jr.
158. What is the remainder when 162. Which of the following angles in standard
534,214,557,989,215 is divided by 4? position is coterminal with 40o?
A. 0 B. 1 C. 2 D. 3 A. 2200o B. 1760o
C. 1520o D. 1360o
Explanation:
The divisibility rule for 4 tells us that our Explanation:
concern would only be the last 2 digits. Coterminal angles are congruent, modulo 360.
15 ÷ 4 = 3 r. 3 That means they will leave the same remainder
when divided by 360.
In textbooks, θ is coterminal with any angle
159. Dividing by 0.125 is the same as multiplying expressed as 360N + θ wherein N is an integer.
by which number?
To easily tackle this question, simply subtract 40
A. 5 B. 8 C. 10 D. 16 from each of the choices, then see if any of those
is divisible by 360 (or leaves a remainder of 0
when divided by 360) using your calculator.
Explanation:
2200 – 40 = 2160; 2160 ÷ 360 = 6
Just use 1 as your test number.
1÷ 0.125 = 8 .: 2200o is coterminal with 40o

160. Find the surface area of a sphere whose 163. Find the equation of the line passing
radius is 6 cm. through (2,7) and (-3,-3).
A. 72 π cm2 B. 108 π cm2 A. y = 4x -1 B. y = 3x + 1
C. 144 π cm2 D. 192 π cm2 C. y = 3x + 6 D. y = 2x + 3

Solution: Solution:
Surface Area 4 π r2 = 4 (62 π 144 π Two-point form of linear equations:
161. Which of the following is the reference y – y1 = x x
angle of 216o?
y–7= x 2
A. 84o B. 66o C. 54o D. 36o
y – 7 = 2(x – 2)
y – 7 = 2x – 4
Explanation: y = 2x + 3
The reference angle for angles from the different
quadrants are as follows: 164. In which quadrant can we find θ if tan θ < 0
QI: the angle θ itself and sin θ > 0?
QII. 180 – θ A. First Quadrant
QIII. θ – 180 B. Second Quadrant
QIV. 360 – θ C. Third Quadrant
D. Fourth Quadrant

Author: Victor A. Tondo Jr.


Author: Victor A. Tondo Jr.
Solution:
Explanation: a3/2 – 1 = 7
a3/2 = 8
Use the CAST
(a3/2 = 23)2/3
mnemonic.
a = 22 = 4

168. Which of the following is true?


A. A rectangle is a square.
165. Find the equation of the line passing B. A rhombus is a rectangle.
through the point of origin and (3,4). C. A trapezoid is a rhombus.
D. A square is a rhombus.
A. y = x B. y= x

C. y = x + D. y = x + 1
169. What is the measure of each exterior angle
of a pentagon?
Solution: A. 108o B. 72o
C. 60o D. 36o
y – y1 = x x ; (0,0) and (3,4)
y–0= x 0
Solution:
y= x
MEA = 360/N = 360/5 = 72o

166. Find the range of f(x) = -2x2 + 4x. 170. How many diagonals does a nonagon have?
A. y 2 B. y 2 A. 27 B. 36 C. 45 D. 54
C. y -2 D. y -2

Solution:
Explanation:
Diagonals = N(N-3)/2 = 9(6)/2 = 27
Since this is a quadratic function, you need to
know two things to determine its range: its
opening and k of its vertex (h,k). 171. What is the fractional equivalent of
0.123123123123…?
The parabola opens downward since a = -2. A. B. C. D.
k = c – (b2/4a) = -16/(-8) = 2 Algebraic Solution:
Since the parabola opens downward, the graph Let x 0.123123123123…
starts from - , going to k which is 2. Thus, y 2.
1000x 123.123123123123…
1000x – x = 123
167. If a3/2 – 1 = 7, what is a? 999x = 123
A. 4 B. 8 C. 9 D. 18 x = 123/999 or 41/333

Author: Victor A. Tondo Jr.


Author: Victor A. Tondo Jr.
Alternative Solution: 175. This is located at the intersection of the
angle bisectors of a triangle.
Write a fraction whose numerator is the
repeated digits (123) and whose denominator A. Incenter B. Circumcenter
has the same number of digits but is made of 9s C. Centroid D. Orthocenter
(123 is 3-digit, so use 999). Thus, 123/999 or
41/333.
Explanation:
Incenter: intersection of angle bisectors
172. Mrs. Pasay saved P250 after buying a phone Circumcenter: intersection of perpendicular
with a 10% discount. How much did she pay for bisectors
the phone? Centroid: intersection of medians
A. P2500 B. P2250 Orthocenter: intersection of altitudes
C. P2000 D. P1750

176. ∆ABC is similar to ∆DEF. ̅̅̅̅


AB is 9 cm long
Solution: ̅̅̅̅ is 12 cm long. If the area of ∆ABC is
while DE
P250 = discount, 10% = discount rate 27 cm2, what is the area of ∆DEF?
Original Price (OP) = ??? Selling Price = ??? A. 36 cm2 B. 48 cm2
DC = OP x DC Rate C. 60 cm2 D. 72 cm2
250 = OP x (0.1)
250/0.1 = OP
Solution:
2500 = OP
Ratio of sides = 9:12 or 3:4
Selling Price = OP – DC = 2500 – 250 = P2250
Ratio of areas = 32:42 or 9:16
9:16 = 27:N
173. A book was sold for P270 after a 10%
16(27) = 9N; 48 = N
discount was given. How much was the book
originally?
A. P330 B. P300 177. Find the remainder when x4 – 3x3 + 2x2 +
C. P297 D. P280 3x – 9 is divided by (x-3).
A. -18 B. -9 C. 9 D. 18
Solution:
SP = OP (1-DC Rate) Solution:
270 = OP (0.9) Use the remainder theorem.
270/0.9 = OP x-a = x-3  a=3
300 = OP remainder = f(3) = 34 – 3(33) + 2(32) + 3(3) – 9
174. Find the area of an equilateral triangle = 81 – 81 + 18 + 9 – 9 = 18
whose sides measure 12 cm each.
A. 36√3 cm2 B. 48√3 cm2 178. Which of the following has its incenter,
C. 60√3 cm2 D. 72√3 cm2 circumcenter, centroid, and orthocenter in just
one point?

Solution: A. Right Triangles B. Equilateral Triangles


C. Isosceles Triangles D. Scalene Triangles
√ √
AreaEqTri = = = 36√3
Author: Victor A. Tondo Jr.
Author: Victor A. Tondo Jr.
179. Dexter is twice as heavy as Pablo. Ming is Explanation:
4kg heavier than Pablo. The sum of their masses Altitude: perpendicular to one side, passing
is 164kg. How heavy is Dexter? through the opposite vertex.
A. 40 kg B. 44 kg Median: line segment from midpoint of one side
C. 80 kg D. 88 kg to opposite vertex
Bisector: line segment that bisects an angle of a
Solution: triangle
D = 2P, M=P+4
D + P + M = 164 183. How many ways can Lola Leonor arrange
2P + P + (P+4) = 164 her six meals on the Lazy Susan (the rotating
4P + 4 = 164 circular wooden server on top of the table)?
4P = 160; P = 40, D=2(40) = 80 A. 720 B. 120 C. 36 D. 30

180. A circle is drawn inside a triangle such that Explanation:


it is tangent to the sides of the triangle. Its center
will be the triangle’s ___________________. This problem is about Circular Permutations or
arrangements on a circle. The formula is (N-1)!.
A. Incenter B. Circumcenter
C. Centroid D. Orthocenter
184. In parallelogram
MATH, m M = 7x – 12 and
Explanation: m T = 5x + 32. Find m A.
If the circle is inside the triangle, its center is the A. 22 B. 38 C. 44 D. 142
INcenter. If the circle circumscribes the triangle,
its center is the CIRCUMcenter.
Solution:
181. Rayon can do a job in 3 hours, while Carlyn M and T are opposite angles,
can do the same job in 7 hours. How long will it therefore m M = m T.
take them to finish the job by working together? 7x – 12 = 5x + 32
A. 2.1 hours B. 2.5 hours 7x – 5x = 32 + 12
C. 5 hours D. 10 hours 2x = 44; x = 22
.: m M = 7(22)-12 = 154-12 = 142
Solution: M and A are consecutive angles,
therefore m M + m A = 180.
= = 2.1
142 + m A = 180
m A = 180 – 142 = 38
182. This line is perpendicular to one side of the
triangle passing through the opposite vertex.
185. Find the equation of the line perpendicular
A. Longitude B. Median to 2x + 5y = 7, passing through (1, 2).
C. Altitude D. Bisector
A. 2x + 5y = 12 B. 2x – 5y = -8
C. 5x + 2y = 9 D. 5x – 2y = 1

Author: Victor A. Tondo Jr.


Author: Victor A. Tondo Jr.
Solution: 188. What is formed by the intersection of two
planes?
Just like what we did in item #76, simply
interchange the numerical coefficients of x and y A. a point B. a line
in the original equation, then change the C. a plane D. space
operation between them. For the constant,
simply substitute the x and y values of the point
and solve. 189. What is formed when a plane intersects a
cone parallel to its circular base?
A. ellipse B. hyperbola
186. How many ways can the letters of the word C. circle D. parabola
BANANA be rearranged?
A. 720 B. 240 C. 120 D. 60
Explanation:
The conic sections are formed by intersecting a
Solution: cone with a plane.
This is a permutation with repeated elements: Parallel to its base: circle
! Perpendicular to its base: parabola
P = ! ! !… where n is the total number of
Slanted relative to the base: ellipse
elements or letters and a!, b!, c!, … are the
number of times the different elements (or
letters) were repeated. 190. In which non-Euclidean model for geometry
BANANA has 6 letters: 1 B, 3 A, and 2 N can we have any given line ℓ and a point A which
! is not on ℓ, wherein all lines through A will
P= 60 intersect ℓ?
! ! !

A. hyperbolic B. elliptic
C. Saccheri D. Pythagorean
187. “The temperature in Baguio City is 20o
while the temperature in Tuguegarao City is
40o”. What level of data is temperature in Explanation:
degrees Celsius?
In Euclidean geometry, only one line will pass
A. Nominal B. Ordinal through A. In elliptic geometry, all lines will pass
C. Interval D. Ratio through A and intersect ℓ.

Explanation: 191. Which numerical system is sexagesimal


Since you cannot infer that Tuguegarao City is (base-60)?
TWICE AS HOT as Baguio City, then the data is A. Mayan B. Roman
not ratio. C. Babylonian D. Hindu-Arabic
Remember: Temperature in degrees Celsius or
Fahrenheit is interval, but temperature in Kelvin
is ratio. Explanation:
Mayans: base-20 (vigesimal)
Babylonians: base-60 (sexagesimal)
Romans and Hindu-Arabic: base-10 (decimal)

Author: Victor A. Tondo Jr.


Author: Victor A. Tondo Jr.
192. Which numerical system makes use of dots 196. Which of the following has an undefined
and horizontal lines, and shell shapes for zero? slope?
A. Egyptian B. Roman A. a vertical line
C. Greek D. Mayan B. a horizontal line
C. a line parallel to the x-axis
D. a diagonal line
193. Which of the following is false?
A. sin2 θ + cos2 θ 1
Horizontal line (parallel to x-axis): m = 0
B. sin θ csc θ 1
C. sin θ ÷ cos θ tan θ Vertical line (parallel to y-axis): m is undefined
D. sin θ tan θ cos θ Slanted downwards to the right: m is negative
Slanted upwards to the right: m is positive
Explanation:
tan ; or sin θ = tan θ (cos θ) 197. In solid geometry, what do you call a solid
bound by polygons?
A. multigon B. tessellation
194. If three-fourths of a number is 33 more than
C. porygon D. polyhedron
its one-fifth, what is that number?
A. 240 B. 120
C.90 D. 60 198. Tchr. Victor needs to randomly get 10 out of
his 50 students for drug testing. He proceeds by
making the students count off from 1 to 5. He
Solution: then randomly picks a number from 1 to 5.
Which sampling method did he use?
x x + 33
A. stratified B. cluster
20 ( x x + 33)  15x = 4x + 660 C. systematic D. convenience
11x = 660
x = 60
Explanation:
Stratified: /strata/ population has hierarchy or
195. Which of the following has the greatest sub-classifications
value:
Cluster: for homogenous population in a large
A. 3 + 32 + (3 + 3)2 area
B. 33
Systematic: counting off
C. [(3 + 3)2]2
D. (3 + 3 + 3)2
199. Which statistical test must be used in
Solution: testing the significance of group differences
between 2 or more groups?
A. 3 + 32 + (3 + 3)2 = 3 + 9 + 36 = 48
B. 33 = 27 A. Chi Square B. t-test
C. [(3 + 3)2]2 = 362 = 1296 C. ANOVA D. Pearson R
D. (3 + 3 + 3)2 = 92 = 81

Author: Victor A. Tondo Jr.


Author: Victor A. Tondo Jr.
200. Which Mathematician is famous for the 204. Which of the following is not a triangle
Fibonacci sequence? congruence postulate?
A. Ptolemy A. SAS B. ASA C. SAA D. AAA
B. Leonardo Pisano Bigollo
C. Pierre de Fermat
D. Luca Pacioli Explanation:
AAA is a triangle similarity postulate.
201. Which Mathematician is famous for his last
theorem? 205. If A is at (-8,5) and B is at (4,-11), find C if C
A. Pythagoras B. Isaac Newton is three-fourths the way from A to B.
C. Daniel Bernoulli D. Pierre de Fermat A. (1, -7) B. (-4, 1)
C. (1, 1) D. (-4, -7)
202. Which of the following is a square?
A. Polygon ABCD which has 4 congruent sides. Solution:
B. Polygon MATH which has 4 perpendicular Since C is three-fourths the way from A to B, then
sides. its coordinates are:
C. Quadrilateral HEAD which has one pair of
congruent perpendicular bisecting diagonals. x = -8 + [4 – (-8)] = -8 + 9 = 1
D. Quadrilateral FROG which has 4 right angles.
y = 5 + (-11 – 5) = 5 + (-12) = 7

Explanation:
A. ABCD is a rhombus 206. CPCTC stands for “____________ parts of
congruent triangles are congruent”.
B. MATH is a rectangle
D. FROG is a rectangle A. collinear B. complementary
C. corresponding D. conjugate

203. Which of the following is the set of points


whose sum of distance to two fixed points is 207. Victor deposited an amount of P200,000 in
constant? a bank that offers 5% interest compounded per
A. parabola B. circle annum. How much will he have in his account
C. ellipse D. hyperbola after 3 years?
A. P230,000 B. P231,525
C. P23,3050 D. P234,575
Explanation:
Parabola: set of points equidistant to a fixed
point (focus) and a fixed line (directrix) Solution:
Since the interest is compounded annually,
Circle: set of points equidistant to a fixed point
Acct = Principal x (1 + rate)time
(center)
Acct = 200,000 x (1.053)
Ellipse: set of points equidistant to two fixed
points (foci) Acct = 200,000 x 1.157625 = 231,525

Hyperbola: set of points whose difference of


distances to two fixed points is constant

Author: Victor A. Tondo Jr.


Author: Victor A. Tondo Jr.
208. Find the remainder when the polynomial 211. The hypotenuse of a right triangle measures
x4 – 3x3 + 2x2 – 5x + 8 is divided by (x – 3). 40 cm. Find its area if one angle measures 30o.
A. 5 B. 8 C. 11 D. 14 A. 100√3 cm2 B. 200√2 cm2
C. 200√3 cm2 D. 400√2 cm2
Solution:
Use the Remainder Theorem. Solution:
Our divisor is (x – 3), so a = 3. Since the hypotenuse is 40 cm, then the leg
f(3) = 34 – 3 (33) + 2 (32) – 5(3) + 8 opposite 30o is 20 cm (half the hypotenuse), and
f(3) = 81 – 81 + 18 – 15 + 8 = 11 the leg opposite 60o is 20√3 cm (√3 times the
short leg). The area of a right triangle is given by
.
209. What is 60% of 120?

A. 50 B. 72 C. 180 D. 200 200√3 cm2

Solution: 212. Nine cans of soda and four hamburgers cost


60% of 120 translates to (0.60) x 120, or 72. a total of P257. Five cans of soda and seven
hamburgers cost a total of P224. How much is a
can of soda?
210. What percent of 80 is 55? A. P17 B. P19 C. P21 D. P23
A. 145.45% B. 135%
C. 68.75% D. 44%
Solution:
Let C = price of a can of soda,
Solution: B = price of a hamburger
Identify the Part (in textbooks, they use the word 9C + 4B = 257 x7  63C + 28B = 1799
“Percentage” , the Base, and the Rate when you 5C + 7B = 224 x4  20C + 28B = 896
face questions like this. Part comes with the
word “is”, Base comes with the word “of”, while 63C + 28B = 1799
Rate comes with the word “percent” or the 20C + 28B = 896
percent symbol (%). 43C = 903
The formulas are: 43 43
C = 21
Rate: R = x 100%
Part: P = B x R (convert rate to decimal first)
213. The product of two consecutive even
Base: B = (convert rate to decimal first) numbers is 728. What is the smaller number?
We are looking for the Rate in this problem, so A. 22 B. 24 C. 26 D. 28
R = x 100% = 0.6875 x 100% = 68.75%
Solution:
Let x = smaller number;
x + 2 = larger number

Author: Victor A. Tondo Jr.


Author: Victor A. Tondo Jr.
x (x+2) = 728 217. When a number is increased by 3, its square
x2 + 2x = 728; increases by 111. By what does its square
x2 + 2x + 1 = 729 increase when the number is increased by 6?
√x + 2x + 1 √729 A. 222 B. 240 C. 444 D. 480
x+1 = 27
x = 26
Solution:

214. What time is 219 minutes past 6:40 AM? Let’s find the original number first.

A. 8:59 AM B. 9:19 AM (x+3)2 – x2 = 111


C. 9:49 AM D. 10:19 AM x2 + 6x + 9 – x2 = 111
6x + 9 = 111
6x = 102
Solution: x = 17
219 minutes = 3 hrs 39 mins 172 = 289; (17+6)2 = 232 = 529
6:40 529 – 289 = 240
+ 3:39
9:79 or 10:19
218. How many prime numbers are there from 1
to 100?
215. Find the vertex of y = 3x2 – 2x + 11. A. 23 B. 24 C. 25 D. 26
A. ( , ) B. ( , )
C. ( , ) D. ( , ) 219. Find the range of f(x) = 2x2 – 8x + 9.
A. y 0 B. y 1
C. y 9 D. y
Solution:
The vertex is at (h,k) where
Solution:
h= ; k=c-
This is a quadratic function so the graph is a
parabola opening upwards (since A = 2). Solve
216. After getting a 20% discount, Mr. Lopez for k to find its minimum value.
paid P4,000 for a gadget. How much was its k = c – b2/4a = 9 – 64/8
original price? k=9–8=1
A. P4,800 B. P5,000 Therefore, y 1.
C. P8,000 D. P20,000

220. Find the domain of y =


Solution:
Sell Price = Orig Price x (1 - Disct Rate) A. x ±7, ±10 B. x ±7
C. x ±10 D. x 1
4000 = OP x (1 – 0.2)
4000 = OP x (0.8)
4000 ÷ 0.8 = OP Explanation:
5000 = OP The denominator should not be equal to 0.

Author: Victor A. Tondo Jr.


Author: Victor A. Tondo Jr.
221. Solve for x: (x+3)2 = (x-4)2. Solution:
A. x = 0 B. x = ½ n= +1
C. x = 1 D. no solution
n= +1 = 22
Solution:
Sum = (n)
(x+3)2 = (x-4)2
x2 + 6x + 9 = x2 – 8x + 16 Sum = 22 = 1419
6x + 8x = 16 – 9
14x = 7
x=½ 225. Mr. G sold 80% of his apples and still had
213 apples left. How many apples did he have
originally?
222. The diagonal of a rectangular prism is 13
cm long. If it is 3 cm thick and 12 cm long, how A. 1704 B. 1065
wide is it? C. 852 D. 293
A. 3 cm B. 4 cm
C.4√3 cm D. 5 cm Solution:
213 = 20% x Original number of apples
Solution: 213 ÷ 0.2 = Original number of apples = 1065
Diagonal2 = Length2 + Width2 + Height2
132 = 122 + W2 + 32 226. When a number is increased by 4, its square
169 = 144 + W2 + 9 also increases by 168. What is this number?
169 – 144 – 9 = W2
A. 15 B. 19 C. 23 D. 27
16 = W2; or 4 = W

Solution:
223. Which of the following is not a function?
(x+4)2 – x2 = 168
A. y = x2 + 2017x – 2017
x2 + 8x + 16 – x2 = 168
B. y = |2017x| - 2017
8x + 16 = 168
C. y = √2017 + 2017 8x = 152
D. y2 = x + 2017 x = 19

Explanation: 227. Solve for k to make a perfect square


When y is raised to an even number, it trinomial: 9x2 + kx + 25
automatically becomes not a function. A. 10 B. 15 C. 20 D. 30

224. 12 + 17 + 22 + 27 + … + 117 _____ Solution:


A. 1409 B. 1414 9x2 + kx + 25 = (3x)2 + 2(3x)(5) + 52
C. 1419 D. 1424
= (3x)2 + 30x + 52

Author: Victor A. Tondo Jr.


Author: Victor A. Tondo Jr.
228. Find the y-intercept of 2x + 3y = 4. Solution:
A. B. C. D. 2 Convert it to its slope-intercept form.
3x + 5y = 7  5y = -3x + 7
y= x+
Solution:
y-intercept is taken when x = 0. .: m =
2(0) + 3y = 4
3y = 4
232. Which of the following is a polynomial?
y=
A. √3 + 4 + 2 B. 2x + 3√
C. +3 D. √3 x + 7
229. Which of the following points is on the line
y = 2x + 5?
A. (1, 3) B. (2, 9) Explanation:
C. (0, 10) D. (3, 10) A polynomial accepts only WHOLE numbers as
exponents of the variable/s. Only D has whole
numbers as exponents of x.
Explanation:
Just substitute the x and y-values of each point
and see which one makes a true equation. 233. What is the degree of the polynomial
9x4 + 5x3 – 2x2 + 3x – 17?
A. (1, 3)  3 = 2(1) + 5 False
B. (2, 9)  9 = 2(2) + 5 True A. 4 B. 5 C. 9 D. 10
C. (0, 10)  10 = 2(0) + 5 False
D. (3, 10)  10 = 2(3) + 5 False
Explanation:
Degree refers to the highest exponent or sum of
230. Find the intersection of y = -2x + 1 and exponents of the variables in any term of a
y = 3x + 16. polynomial.
A. (-3, 7) B. (-4, 9)
C. (3, -7) D. (4, -9)
234. log2 32√2 = __________.
A. 2.5 B. 3.5 C. 4.5 D. 5.5
Solution:
y = -2x + 1
Solution:
(-) y = 3x + 16
0 = -5x – 15 32 = 25 and √2 = 21/2
5x = -15 5 + ½ = 5.5
x = -3; y = -2(-3) + 1
y = 6+1 = 7

231. Find the slope of 3x + 5y = 7.


A. B. C. D.

Author: Victor A. Tondo Jr.


Author: Victor A. Tondo Jr.
235. If y = √3 + 6 , what is x in terms of y? 238. Find the altitude to the hypotenuse of a
right triangle whose sides measure 5 cm, 12 cm,
A. x = √ –1 B. x = √ +1 and 13 cm.
A. B. C. D. 26
C. x = √ +1 D. x = √ –1

Solution:
Solution:
Altitude to the Hyp = (L1 L2)/Hyp = 5(12)/13
y = √3 +6 Altitude to the Hyp = 60/13
Square the equation y2 = 3x2 + 6x
Divide by 3 = x2 + 2x 239. Find the slope of the line tangent to
y = x3 – 6x2 + 2x + 7 at x = 4.
Complete the square + 1= x2 + 2x + 1
A. -8 B. -2 C. 2 D. 8
= x2 + 2x + 1

Extract the root √ =x+1 Solution:

Finally, √ - 1= x Get the first derivative of y.


y' = 3x2 – 12x + 2
Substitute x 4 for every x in y’.
236. Which of the following is a pair of parallel
lines? 3(4)2 – 12(4) + 2 = 48 – 48 + 2 = 2

A. y = 2 and x = 2
B. 12x + 13y = 14 and 13x + 14y = 15 240. Find the average rate of change of
C. y = 3x + 8 and 3y = x + 9 y = x3 – 2x + 3 from x = 0 to x = 3.
D. 4x + 5y = 6 and 8x + 10y = 21
A. 5 B. 6 C. 7 D. 8

Explanation:
Solution:
Parallel lines have the same slope. Both lines
have a slope of -4/5 in D. Average Rate of Change = = 7

237. Which of the following is a pair of 241. Find the radius of x2 + y2 + 2x – 4y = 44.
perpendicular lines?
A. √39 B. 2√11 C. 7 D. 3√6
A. x = 5 and y = 7
B. y = x and 2y = 4x + 5
C. x = 2y + 3 and 2x + 3y = 4 Solution:
D. y = 5x + 6 and y = 0.2x – 8
Complete the squares on the left side of the
equation to return it to its center-radius form.
Explanation: x2 + y2 + 2x – 4y = 44  x2 + 2x + y2– 4y = 44
Perpendicular lines have negative reciprocal (x2 + 2x + 1)+ (y2– 4y +4)= 44 + 1 + 4
slopes. For choice A, x=5 is horizontal and y=7 is (x2 + 2x + 1)+ (y2– 4y +4)= 49
vertical, therefore they are perpendicular.
Author: Victor A. Tondo Jr.
Author: Victor A. Tondo Jr.
242. Gian has 8 more P5 coins than P1 coins. If 245. Find the mode of the following scores:
he has a total of P106, how many P5 coins does 78 78 78 78 79 79 79
he have? 79 80 80 80 80
A. 13 B. 15 C. 17 D. 19 A. 79 B. 78, 79, and 80
C. 80 D. no mode
Solution:
Let x = number of P1 coins Explanation:
.: x+8 = number of P5 coins Mode, by definition, is the score with the highest
1(x) + 5 (x+8) = 106 frequency. Since each score has a frequency of 4,
x + 5x + 40 = 106 then there is no mode.
6x + 40 = 106
6x = 66
x = 11 246. The average grade of 23 students in Section
A is 86, while the average grade of 27 students in
.: He has 11 P1 coins and (11+8) or 19 P5 coins. Section B is 91. What is the average grade of all
50 students in both sections?
243. After using half of her budget on bills, one-
third on groceries, and P270 on a shirt, Mrs. D A. 88.5 B. 88.6 C. 88.7 D. 88.8
still had P130 left. How much was her budget?
A. P2400 B. P2700 Solution:
C. P3000 D. P3300
Average = 88.7
Solution:
Let x = budget
247. Find the axis of symmetry of y = 3x2 – 5x.
x + x + 270 + 130 = x
A. x B. x =
x + 400 = x
C. x = D. x =
400 = x – x
400 = x Explanation:
2400 = x The axis of symmetry is located at x = .
244. x varies directly as y and inversely as z. If
x = 24 when y = 32 and z = 4, what is x when
248. Find the range of the following scores:
y = 21 and z = 7?
19 25 24 31 23 29 33
A. 3 B. 5 C. 7 D. 9 A. 12 B. 13 C. 14 D. 15
Solution:
x = ky/z 24 = k(32)/4 Explanation:
24 = 8k
3=k Range = Highest Score – Lowest Score
x = 3y/z 249. Mr. C travels for 2 hours at a speed of 38
x = 3 (21) / 7 kph and then north for 3 hours at a speed of 53
x = 63/7 = 9 kph. What is his average speed?
A. 44 kph B. 45.5 kph
C. 47 kph D. 48.5 kph

Author: Victor A. Tondo Jr.


Author: Victor A. Tondo Jr.
Solution: 252. What conic figure does the equation
x2 + y2 + 8x – 6y = -100 form?
Average Speed =
A. Real circle B. Degenerate circle
Average Speed = 47 kph C. Imaginary circle D. Ellipse
Solution:
Use CTS (completing trinomial squares) to
250. Victor, Chris, and Diana volunteered to convert the equation to its center-radius form.
teach at a nearby daycare. Chris worked for 2 x2 + y2 + 8x – 6y = -100
hours less than Diana. Victor worked twice as (x2 + 8x + 16) + (y2 – 6y + 9) = -100 + 16 + 9
many hours as Chris. Altogether, they worked for (x + 4)2 + (y – 3)2 = -75
58 hours. How many hours did Victor work?
A. 14 B. 16 C. 28 D. 32 Since r2 = -75, then the radius is imaginary,
making it an imaginary circle.

Solution:
Diana = n hours 253. Find the center of x2 + y2 + 6x – 10y = 2.
.: Chris = (n – 2) hours A. (6, -10) B. (-6, 10)
.: Victor = 2(n-2) hours C. (-3, 5) D. (3, -5)
n + (n-2) + 2(n-2) = 58
n + n - 2 + 2n - 4 = 58 Solution:
4n – 6 = 58 The center of x2 + y2 + Dx + Ey + F = 0
4n = 64 is at ( , .
n = 16
.: Victor worked for 2(16-2) or 28 hours.
254. Find the equation of the circle with center at
(2, 3), passing through (5, -1).
251. What conic figure does the equation A. x2 + y2 + 4x + 6y = 0
x2 + y2 + 4x = -4 form? B. x2 + y2 + 4x + 6y = 12
A. Real circle B. Degenerate circle C. x2 + y2 – 4x – 6y = 0
C. Imaginary circle D. Ellipse D. x2 + y2 – 4x – 6y = 12

Solution: Solution:

Use CTS (completing trinomial squares) to Find the radius first by using the center-radius
convert the equation to its center-radius form. form.
x2 + y2 + 4x = -4 (x – 2)2 + (y – 3)2 = r2
(x2 + 4x + 4) + y2 = -4 + 4 (5 – 2)2 + (-1 – 3)2 = r2
(x + 2)2 + y2 = 0 9 + 16 = r2
25 = r2
Since r2 = 0, then the radius is also 0, making it a 5=r
degenerate circle.
Substitute this to your center-radius form and
simplify.
(x – 2)2 + (y – 3)2 = 52
x2 – 4x + 4 + y2 – 6y + 9 = 25
x2 + y2 – 4x – 6y + 13 = 25
x2 + y2 – 4x – 6y = 12
Author: Victor A. Tondo Jr.
Author: Victor A. Tondo Jr.
255. Find the equation of the vertical line passing Solution:
through (-3, 4).
A. x = -3 B. x = 4 Use the midpoint formula: xm = , ym =
C. y = -3 D. y = 4
2= -3 =
Explanation: 4 = (-4) + x2 -6 = 5 + y2
Equation of vertical lines: x = abscissa of your 8 = x2 -11 = y2
point

259. The endpoints of the diameter of a circle are


256. Find the equation of the horizontal line A(9, -5) and B(-3, 11). What is the equation of
passing through (-3, 4). the circle?
A. x = -3 B. x = 4 A. (x – 3)2 + (y – 3)2 = 100
C. y = -3 D. y = 4 B. (x – 3)2 + (y – 3)2 = 400
C. (x + 3)2 + (y + 3)2 = 100
Solution: D. (x + 3)2 + (y + 3)2 = 400
Equation of horizontal lines: y = ordinate of your
point Solution:
Find the midpoint of the diameter to get the
center, C(h,k).
257. Which of the following lines passes through
the point (3, -2)? h= k=
A. y = x + 5 B. y = 2x – 8 h = 3, k = 3
C. y = 5 – x D. y = 5 – 2x
Next, find the length of the radius. You can do
Solution: this by either A) getting half of the length of the
diameter AB, or B) using the center-radius form
Simply substitute x and y from your point. If the of the circle. The second option would be more
equation holds true, then the given line passes preferable as it already gives us the equation of
through your point. the circle.
y=x+5  (-2 3+5 (x – 3)2 + (y – 3)2 = r2
y = 2x – 8  (-2) = 2(3) – 8 Using the point A(9, -5), we get
y=5–x  (-2 5-3 (9 – 3)2 + (-5 – 3)2 = r2
y = 5 – 2x  (-2 5 – 2(3) 36 + 64 = r2
100 = r2

258. Given that I(2, -3) is the midpoint of V(-4, 5) .: (x – 3)2 + (y – 3)2 = 100
and C, find the coordinates of C.
260. Find the distance between the line
A. (-1, 1) B. (1, -1)
3x + 4y – 5 = 0 and the point (8, -1).
C. (8, -11) D. (-10, 16)
A. 2 B. 3 C. 4 D. 5

Author: Victor A. Tondo Jr.


Author: Victor A. Tondo Jr.
Solution: 9 2 5
A∆ = ½ * +
3 8 1
Use the formula for distance of point (x1, y1) A∆ = ½ [(-72 + -2 + -15) – (6 + 40 + -9)]
from line Ax + By + C = 0: D = √ A∆ = ½ |(-89 – 37)|
A∆ = ½ (126) = 63
D= √
D= =3
262. Which of the following is outside the circle
261. B is one-fourth of the way from A(-13,9) to defined by the equation (x – 3)2 + y2 = 40?
C(7,-7). Find the coordinates of B. A. (5, 6) B. (7, 5)
C. (0, -5) D. (-1, 4)
A. (2, -3) B. (-8, 5)
C. (2.5, -3.5) D. (-7.5, 5.5) Solution:

Solution: A point is outside the circle when its distance


from the center is greater than the radius.
In this question, it is important to note the A: (5, 6)  (5 – 3)2 + 62 = 40
“from” point and the “to” point. This point is ON the circle.
Abscissa of point B = abscissa of “from” point + B: (7, 5)  (7 – 3)2 + 52 > 40
¼ abscissa of “to” point minus abscissa of This point is OUTSIDE the circle.
“from” point
C: (0, -5)  (0 – 3)2 + (-5)2 < 40
Abscissa of point B = -13 + ¼ (7 – (-13)) This point is IN the circle.
= -13 + ¼ (20) = -8
D: (-1, 4)  (-1 – 3)2 + 42 < 40
Ordinate of point B ordinate of “from” point + This point is IN the circle.
¼ ordinate of “to” point minus ordinate of
“from” point)
263. Which of the following is parallel to the line
Ordinate of point B = 9 + ¼ (-7 – 9) defined by the equation y = 3x – 4?
= 9 + ¼ (-16) = 5 A. y + 3x = 5 B. x + 3y = 6
C. y = x + 7 D. y = 3x + 8
261. Find the area of the triangle whose vertices
are X(-9, -3), Y(-2, 8), and Z(5, 1). Solution:
A. 61 B. 62 C. 63 D. 64 Two lines are parallel when their slopes are
equal. Check the slopes of the lines by converting
Solution: them to their slope-intercept form (y = mx + b).
A) y + 3x = 5  y = -3x + 5 m = -3
The biggest mistake you could make in solving
B) x + 3y = 6  y= x+2 m=
this problem is by getting the equation of one the
line formed by one pair of vertices, getting the C) y = x + 7 m=
length of the said side, and getting the distance of D) y = 3x + 8  m=3
the third point from the line you first obtained,
then multiplying the obtained distance by the
length of the line segment and then dividing by 264. Which of the following is perpendicular to
two. That’s a very long solution. the line defined by the equation y = 3x – 4?
A. y + 3x = 9 B. -x + 3y = 10
Simply use the formula A∆ = ½ * + C. y = x + 11 D. y = 3x – 12

Author: Victor A. Tondo Jr.


Author: Victor A. Tondo Jr.
Solution: 267. Which of the following equations pertain to
a parabola that opens to the right?
Two lines are perpendicular when their slopes
are negative reciprocals of each other, or when A. 4(y + 3) = (x – 2)2
m1 x m2 = -1. The slope of the given line is 3, so B. -3(y – 4) = (x + 5)2
the slope of the other line is . Check the slopes C. (y – 6)2 = 5(x + 1)
D. (y + 8)2 = -2(x – 3)
of the lines by using their slope-intercept form
(y = mx + b).
Solution:
A) y + 3x = 9  y = -3x + 9 m = -3 The parabola opens to the right when the
B) -x + 3y = 10 y= x+ m= squared variable is y and the coefficient of x is
positive. It opens to the left when the squared
C) y = x + 11 m= variable is y and the coefficient of x is negative.
D) y = 3x – 12  m=3

268. Which of the following equations pertain to


265. Which of the following is coincidental to the a parabola that opens downward?
line defined by the equation y = 2x + 13?
A. 4(y + 3) = (x – 2)2
A. y + 2x = 13 B. 2x – y + 13 = 0 B. -3(y – 4) = (x + 5)2
C. y = x + 13 D. 2y = 2x + 13 C. (y – 6)2 = 5(x + 1)
D. (y + 8)2 = -2(x – 3)
Solution:
Solution:
Two lines are coincidental when their equations
are equivalent to each other, or when their The parabola opens upward when the squared
slopes and y-intercepts are equal. variable is x and the coefficient of y is positive. It
opens to the downward when the squared
A) y + 2x = 13  y = -2x + 13 variable is x and the coefficient of y is negative.
B) 2x – y + 13 = 0  y = 2x + 13
C) y = x + 13  y = x + 13
D) 2y = 2x + 13  y=x+ 269. How long is the latus rectum of the parabola
defined by 12(y – 4) = (x + 3)2?
A. 12 B. 6 C. 4 D. 3
266. Which of the following equations pertain to
a parabola? Solution:
A. y2 + 5y = x In the form 4A(y – k) = (x – h)2, the length of the
B. x2 + y2 + 3x – 4y = 0 latus rectum is 4A.
C. + 1
D. 1 270. How far is the vertex from the directrix of
the parabola defined by 16y = x2?
Solution: A. 16 B. 8 C. 4 D. 2
A parabola’s equation includes two variables, Solution:
one of which is squared.
The distance of the vertex from the directrix is A
(from 4A, which is the length of the latus
rectum). The distance of the vertex from the
focus is also A. The vertex is between the focus
Author: Victor A. Tondo Jr.
Author: Victor A. Tondo Jr.
and the directrix, which means from focus to Explanation:
directrix is 2A. The distance from focus to any
Quadrant Abscissa Ordinate
endpoint of the latus rectum is also 2A.
First Positive Positive
Second Negative Positive
271. Find the equation of the directrix of the Third Negative Negative
parabola defined by (y – 2)2 = -4(x + 3). Fourth Positive Negative

A. x = -2 B. x = 2
C. y = -4 D. y = -2 274. Find the distance between the parallel lines
y = 3x + 9 and y = 3x – 12.
Solution:
√ √
A. 7 B. 21 C. D.
To find the directrix,
first get the vertex
and the opening of Solution:
the parabola. The Use the distance between two parallel lines
vertex is at (-3, 2)
D=√
and the parabola
opens to the left, √
which means the D= √
= =

directrix is vertical.
The directrix is A units from the vertex. Since
4A = -4, then A = -1. The directrix is 1 unit to the 275. Find the intersection of the lines y = 2x + 5
right of the vertex (-3, 2), meaning it passes and y = -4x + 23.
through (-2, 2). The vertical line passing through A. (3, 10) B. (3, 11)
(-2, 2) is x = -2. See attached image. C. (4, 10) D. (4, 11)

Solution:
272. Find the coordinates of the focus of the
Here we have a system of linear equations in two
parabola defined by -12(y – 4) = (x + 5)2.
variables, namely x and y. Solve for x and y by
A. (-5, 7) B. (-5, 1) using either substitution or elimination.
C. (-8, 4) D. (-2, 4)
y = 2x + 5 y = -4x + 23
Solution: 2x + 5 = -4x + 23
6x = 18
The parabola opens downward, with vertex at
x=3
(-5, 4). From the equation, 4A = -12, or A = -3.
This means the focus is 3 units downward from
y = 2x + 5 = 2(3) + 5 = 11
the vertex. Therefore, the vertex is at (-5, 1).

276. Which of the following pertains to a circle


273. In which quadrant would G(3,-4) fall?
that is concentric with (x – 3)2 + y2 = 24?
A. First quadrant B. Second quadrant
A. x2 + y2 – 6x + 3 = 0
C. Third quadrant D. Fourth quadrant
B. x2 + y2 + 6y –15 = 0
C. x2 + y2 + 6x – 25 = 0
D. x2 + y2 + 6x + 6y – 24= 0

Author: Victor A. Tondo Jr.


Author: Victor A. Tondo Jr.
Solution: Solution:
Concentric circles have the same center. That Coterminal angles share the same initial side and
means the center we are looking for is (3, 0). terminal sides. Finding coterminal angles is done
A. x2 + y2 – 6x + 3 = 0 Center: (3, 0) by adding or subtracting multiples of 360° or 2π
B. x2 + y2 + 6y –15 = 0 Center: (0, -3) rad to each angle, depending on whether the
C. x + y + 6x – 25 = 0
2 2 Center: (-3, 0) given angle is in degrees or radians.
D. x2 + y2 + 6x + 6y – 24= 0Center: (-3, -3) 503o = 143o + (1)(360o).

277. Which of the following is the equation of the 280. Which of the following is NOT a
parabola that opens upward, whose latus rectum trigonometric identity?
is 12 units long, directrix is y = -3, and line of A. sin2 θ + cos2 θ 1
symmetry is x = 7?
B. tan θ
A. 12y2 = x – 7
C. 1 + tan2 θ sec2 θ
B. 12y = (x – 7)2
D. 1 – cot2 θ csc2 θ
C. 12(y + 3) = (x – 7)2
D. 12(y – 3) = (x – 7)2
Explanation:
Solution: The correct identity goes 1 + cot2 θ csc2 θ.
Since the latus rectum is 12 units long, then
4A = 12, or A = 3. The parabola opens upward,
281. Which of the following is false?
which means the vertex must be 3 units above
the directrix y = -3. Therefore, the ordinate of A. tan θ B. csc θ
the vertex (k) is equal to (-3) + 3 or 0. The line C. cos θ D. sec θ
of symmetry is x = 7, which means that the
abscissa of the vertex (h) is also 7. The parabola Explanation:
opens upward, therefore the equation should be
4A(y – k) = (x – h)2. Just use the SohCahToa – ChoShaCao mnemonic.
The reciprocal of sine is cosecant (csc), the
reciprocal of cosine is secant (sec), and the
278. Convert rad to degrees. reciprocal of tangent is cotangent (cot).

A. 1.309o B. 75o
C. 150o D. 216o 282. Which of the following is NOT a cofunction
identity?
Solution: A. cos θ sin 90 – θ
π rad 180o B. cot (90 – θ tan θ
Therefore rad = (180o) = 75o C. sec θ csc 90 – θ
D. csc θ sin 90 – θ

Explanation:
279. Which of the following angles is coterminal
with 143o? The value of a trigonometric function of an angle
A. 217o B. -37o equals the value of the cofunction of the
C. 323o D. 503o complement of the angle. Cofunctions are sine
and COsine, tangent and COtangent, and secant
and COsecant.

Author: Victor A. Tondo Jr.


Author: Victor A. Tondo Jr.
cos θ sin 90 – θ sec θ csc 90 – θ Solution:
tan θ cot 90 – θ sin θ cos 90 – θ There are 14 multiples of 7 from 1 to 100 since
csc θ sec 90 – θ cot θ tan (90 – θ 100 ÷ 7 = 14 r 2.
Therefore the probability is or .
283. The hypotenuse of a 30-60-90 triangle is 48
cm long. How long is its shortest side?
A. 24 cm B. 24 √2 cm 286. What is the probability of rolling a sum of
10 when rolling two dice?
C. 24 √3 cm D. 16√3 cm
A. B. C. D.
Explanation:
For any 30-60-90 triangle, the ratio of the sides Solution:
are as follows:
Short leg (opposite of the 30o angle) = n The preferred outcomes are (4,6), (5,5), and
(6,4). There are 36 possible outcomes. Therefore
Long leg (opposite of the 60o angle) = n√3
the probability is 3/36 or 1/12.
Hypotenuse = 2n

Since the hypotenuse 2n is equal to 48, then 287. Factorize 3x2 + 5x – 2.


n equals 24. A. (3x – 1) (x + 2) B. (3x + 1) (x – 2)
C. (3x – 2) (x + 1) D. (3x + 2) (x – 1)
284. In a right triangle, the side opposite an
angle measuring 50o is 100 cm long. How long is 288. Six-sevenths of a number is 6 less than nine-
the side adjacent to the 50o angle? tenths of the same number. What is the number?
A. 93.45 cm B. 83.91 cm A. 130 B. 140 C. 200 D. 210
C. 149.14 cm D. 200 cm

Solution: 289. A certain University has a dormitory. If 10


students stay in a room, 24 students will not
The given are an angle and the sides opposite have a room. If 12 students stay in a room, there
and adjacent to it. Use the tangent function. will be 6 vacant beds. How many rooms are there
in the dormitory? How many students are
tan 50o = staying in the dormitory?
A. 116 B. 115 C. 114 D. 113
tan 50o =
x= = 83.91 cm Solution:
Let n = the number of students
and x = the number of rooms
285. A hundred cards are numbered 1 to 100.
What is the probability of drawing a card whose n = 10x + 24 and n = 12x + 6
number is divisible by seven? 10x + 24 = 12x + 6
A. B. C. D. 24 – 6 = 12x – 10x
18 = 2x
9=x
Next, n = 10x + 24 = 10(9) + 24 = 114
Therefore, there are 114 students.
Author: Victor A. Tondo Jr.
Author: Victor A. Tondo Jr.
290. Which of the following is not between 294. Determine which equation is equivalent to
4x – 1 = 11.
and ? A. 4x = 10 B. 3x = 11
A. B. C. D. C. 4 – 1x = 11x D. 4x = 12

Solution:
295. Determine the relation that matches the
= -0.4 and = -0.75. table of values.
x 1 2 3 4 5
A. = -0.8 y 13 11 9 7 5
B. = -0.65 A. y = 21 – x B. y = 15 – 2x
C. y = 3x + 7 D. y = 2x + 11
C. = -0.5
D. = -0.6
296. Determine which polynomial expression
Only A is beyond -0.4 to -0.75. matches the algebra tile model.

291. Which value describes the position of C? A. 2x2 + x + 4 B. 3x2 + x + 4


C. 3x2 – x + 4 D. 3x2 + x – 5

A. -0.75 B. -0.6 C. -1.25 D. -1.4


297. Subtract: (–2x2 + 5x – 9) – (2x – 7)
A. 2x2 + 3x + 16 B. -2x2 + 3x + 2
292. Choose the correct value of (x + y)(x – y) C. -2x + 3x – 2
2 D. -2x2 + 7x + 2
when x = 3.5 and y = –8.7
A. -63.44 B. 63.44
C. -148.84 D. 10.4 298. Evaluate the polynomial 4x2 – 6x – 3 if x = 2.
A. -1 B. 1 C. 3 D. 5
Solution:
Solution:
(x + y)(x – y) = (3.5 + (-8.7))(3.5 – (-8.7))
4x2 – 6x – 3 when x = 2
= (-5.2)(12.2)
= 4(2)2 – 6(2) – 3
= -63.44
=1

293. Which two triangles are similar?


299. Determine the measure of Y and Z.

A. A and B B. A and C
C. B and D D. B and C A. 45o, 55o B. 40o, 50o
C. 48o, 48o D. 50o, 50o
Explanation:
B and D are isosceles right triangles, or 45-45-90 Since Y and Z intercept the same arc as X,
triangles. Since they have congruent then they are congruent to X.
corresponding angles, then they are similar.
Author: Victor A. Tondo Jr.
Author: Victor A. Tondo Jr.
300. Which of the following is equal to x? 303. The graph below represents the motion of a
car. The graph shows us that the car is:

A. 15 sin 72o B. 15 sin 18o


C. 72 sin 72o D. 72 sin 18o
A. accelerating
Solution: B. standing still
C. travelling north-east
x is the adjacent side to the 72o angle and 15 is D. travelling at a constant speed
the hypotenuse. The trigonometric function for
adjacent and hypotenuse is cosine. However,
none of the choices made use of cos 72o. 304. The units digit of the number 543444 is:
Therefore we must use the other angle which A. 3 B. 9 C. 7 D. 1
measures 18o. Its opposite side is x and the
hypotenuse is still 15. Therefore, Solution:
sin 18o =
The last digit of the powers of 3 (the last digit of
15 sin 18o = x 543 are 3, 9, 7, 1, 3, 9, 7, 1, 3, 9, 7, 1, …
The last digit repeats after four powers.
The last digit of 31, 35, 39, … is 3.
301. Find the points of intersection of the graphs The last digit of 32, 36, 310, … is 9.
of y = x2 and y = 3x – 2. The last digit of 33, 37, 311,… is 7.
A. (1, 1) and (1, 4) B. (1, 1) and (2, 4) The last digit of 34, 38, 312, … is 1.
C. (1, -1) and (2, 4) D. (-2, 4) and (1, 1)
Therefore the units digit of 543444 is 1.
Solution:
By transitive property, we can say x2 = 3x – 2. 305. 4n+1 (4n+2) equals
x2 – 3x + 2 = 0 A. 42n + 3 B. 82n + 3
(x – 2)(x – 1) = 0 C. 162n + 3 D. 4n + 3
x = 1, 2
Explanation:
When x = 1, then y = 12 = 1. Apply the laws of exponents.
When x = 2, then y = 22 = 4.
Therefore, the points of intersection of the
graphs are (1,1) and (2,4). 306. The greatest number of Fridays that can
occur in a 75 day period is:
A. 10 B. 11 C. 12 D. 13
302. An approximate value for
. . Explanation:
is:
.
A. 2 B. 20 Let the first day be Friday. The last Friday will be
C. 200 D. 2000 on the 71st day. That’s a total of 11 Fridays.

Explanation:
Ignore the decimals of the large numbers and use
only the first few decimals of 0.0403289925.
Using your calculator, 302476 x 0.04 ÷ 5962 2.
Author: Victor A. Tondo Jr.
Author: Victor A. Tondo Jr.
307. Which model is not a function? 312. A weather station recorded the amount of
rain that fell during an 8-hour time frame using a
rain gauge. The findings are recorded in the
graph below.

A. B.

C. D.

Explanation:
In a function, no x-value can ever have two or Between which hours was the rate at which the
more corresponding y-values. rain fell greater than the rate at which the rain
fell between hours 0 and 1?
A. between hours 3 and 4
308. Which expression is equivalent to (9-2)8? B. between hours 4 and 5
A. -8132 B. -818 C. D. C. between hours 5 and 6
D. between hours 7 and 8
Solution:
(9-2)8 = ( )
312. Each day of the month, Carl earns an
allowance, in cents, equal to the square of that
309. What is 5 × 10–4 written in standard date of the month. Which is a number of cents
notation? Carl could earn in a single day?
A. 0.00005 B. 0.0005 A. 21 B. 31 C. 64 D. 111
C. 5,000 D. 50,000
Explanation:

310. What is the value of 54 × 5-6? In the choices, only 64 is a perfect square.
A. -25 B. C. D. 25
313. Which set of ordered pairs models a
Solution: function?
54 × 5-6 = 5-2 = A. {(2, 9), (7, 5), (3, 14), (2, 6)}
B. {(5, 10), (5, 15), (5, 20), (5, 25)}
C. {(3, 10), (4, 15), (5, 20), (3, 25)}
311. . Which comparison is true? D. {(–10, 20), (–20, 30), (–30, 40), (–40, 10)}
A. 4 < 180.5 < 4.5 B. 4.5 < 180.5 < 5
C. 8.5 < 180.5 < 9.5 D. 17 < 180.5 < 19 Explanation:
Only D has no repeated x-value.
Explanation:
180.5 = √18 4.24

Author: Victor A. Tondo Jr.


Author: Victor A. Tondo Jr.
314. Which number is irrational? 316. Praetor jogged on a path that was 2 miles
A. B. √121 long, took a break, and then jogged back along
̅̅̅̅̅̅̅ the same path to where he started. He jogged at
C. 16.12131415... D. 0.002018
different speeds for different distances along the
path as shown in the graph.
314. Rayon has a piece of rectangular paper that
is 12 inches wide by 16 inches long. He drew a
straight line along the diagonal of the paper.
What is the length of the line Rayon drew?
A. √28 inches B. √192 inches
C. 20 inches D. 28 inches

Explanation:
Use the Pythagorean theorem.
Between which times did Praetor jog the fastest?
A. 0 minutes and 10 minutes
315. Which equation has infinitely many B. 10 minutes and 25 minutes
solutions? C. 25 minutes and 30 minutes
A. 2x + 4 = 7x + 9 D. 30 minutes and 60 minutes
B. 3(2x + 5) = 6x + 15
C. 4x + 13 = 5x + (20 – x) Explanation:
D. x + 3 = 5x – 21
The steepest part of the graph is between 0 and
Explanation: 10 minutes.

A. 2x + 4 = 7x + 9
4 – 9 = 5x – 2x 317. Which expression has a value of -2?
-5 = 3x A. |2| + |-4| B. |-2| – |4|
-5/3 = x  One unique solution C. |4| – |-2| D. |-4| + |2|
B. 3(2x + 5) = 6x + 15
6x + 15 = 6x + 15 Explanation:
0=0  True equation. This A. |2| + |-4| = 2 + 4 = 6
means there are infinitely B. |-2| – |4| = 2 – 4 = -2
many solutions. C. |4| – |-2| = 4 – 2 = 2
C. 4x + 13 = 5x + (20 – x) D. |-4| + |2| = 4 + 2 = 6
4x + 13 = 4x – 20
4x – 4x = -20 – 13
0 = -33  False equation. There is 318. Reion is tossing a six-sided number cube
no solution. labeled 1, 2, 3, 4, 5, and 6. What is the probability
of tossing 6 twice in a row?
D. x + 3 = 5x – 21 A. B. C. D.
3 + 21 = 5x – x
24 = 4x
6=x  One unique solution
319. Which represents the value of x in
6 – 4x 26?
A. x -8 B. x -8
C. x -5 D. x -5
Author: Victor A. Tondo Jr.
Author: Victor A. Tondo Jr.
320. The table below shows the resting heart 324. In the spinner, what is the probability of the
rates in beats per minutes of six students. The arrow NOT landing on the space with the ∆?
rate, 40 beats per minute, seems to be an outlier.
Which measure of central tendency changes the
least by dropping 40 from the data?
Heart
Rate
78 71 79 80 40 71
A. mean B. median
A. B. C. D.
C. mode D. range

Explanation:
325. Which values of x and y make the system of
The original mean is 69.833. When 40 is equations below true?
removed from the data, it becomes 75.8. The 2x - y = -1
mean decreases by 5.967. 3x - y = -3
The original median is 74.5. When 40 is removed A. x = -4; y = -7 B. x =-2; y = -3
from the data, it becomes 78. The median C. x = 2; y = 5 D. x = 4; y = 15
increases by -3.5.
The original mode is 71. When 40 is removed Solution:
from the data, it stays the same.
2x - y = -1
The original range is 40. When 40 is removed (-) 3x - y = -3
from the data, it becomes 9. The range decreases -x = 2
by 31. x = -2
2x - y = -1
2(-2) - y = -1
321. The sum of a number, n, and 5 is subtracted
-4 - y = -1
from 8. Which expression represents this
-4 + 1 = y
statement?
-3 = y
A. 8 – (n + 5) B. (n + 5) + 8
C. (n + 5) – 8 D. 8 + (n + 5)
326. The lengths of two sides of a triangle are 8
inches and 13 inches. Which of the following
322. How is 0.5600 written in scientific notation?
represents x, the possible length in inches of the
A. 5.6 × 10 B. 5.6 × 10-1
remaining side of the triangle?
C. 5.6 × 10 -2 D. 5.6 × 10-3
A. 5 < x < 21
B. 5 x 21
C. x < 5 or x > 21
323. What is the value of x in 3(x – 4) = –21?
D. x 5 or x 21
A. x = –11 B. x = –3
C. x = 3 D. x = 11
Explanation:
Solution: The third side of any triangle must be between
the difference and the sum of the two other
3(x – 4) = –21
sides.
x – 4 = -7
x = -7 + 4 = -3

Author: Victor A. Tondo Jr.


Author: Victor A. Tondo Jr.
327. What is the value of the expression 333. Let A be a set such that A = {v, w, x, y, z}.
below? How many subsets does set A have?
80 ÷ ( 6 + (3 – 5) x 2) A. 5 B. 10 C. 25 D. 32
A. -8 B. 8 C. 10 D. 40
Explanation:
Let n = number of elements
328. Which of the following is closest to the
Number of subsets = 2n
value of the expression below?
(20.0143642359)2 x 8π
A. 1,000 B. 10,000
334. Solve for x:
C. 100,000 D. 1,000,000
2 (5x – 11) + 7 = 3 (x – 7) – 15
A. x = 3 B. x = 1
Explanation:
C. x = -1 D. x = -3
Round off and solve.
202 x 8 x 3.14 = 10,048 Solution:
2 (5x – 11) + 7 = 3 (x – 7) – 15
10x – 22 + 7 = 3x – 21 – 15
329. Which of the following expressions has a
10x –15 = 3x – 36
value of 0?
7x = -21
A. (2 – 3) – (2 – 3) B. (2 – 3) – |2 – 3|
x = -3
C. (2 – 3) + (-3 + 2) D. |2 – 3| – (2 – 3)
335. What is the 4th term in the expansion of
(2x + 3y)7?
330. What is the factorization of 10x2 – x – 21?
A. 15120 x4y3 B. 7560 x4y3
A. (5x – 7) (2x + 3) B. (5x + 7) (2x – 3)
C. 3780 x3y4 D. 1890 x3y4
C. (5x + 3) (2x – 7) D. (5x – 3) (2x – 7)
Solution:
331. Evaluate: (√343)2 The mth term of the expansion of (a + b)n is given
A. 7√7 B. 49 C. 49√7 D. 343 as nC(m-1) an-m+1bm-1.
7C4 (2x)4(3y)3 = 35 (16x4) (27y3)
= 15120 x4y3
332. Find the length of the latus rectum of the
336. The shell shape , as used in the Mayan
ellipse defined by + 1.
numeral system, is the symbol for which
A. B. C. D. number?
A. 100 B. 10 C. 1 D. 0
Solution:
Explanation:
Length of latus rectum of ellipse = The Mayan numerals consisted of only three
Since the lower denominator is 16, then b = 4. symbols: zero, represented as a shell shape; one,
Since the higher denominator is 25, then a = 5. a dot; and five, a bar.
2 2 4 32
5 5 337. Which of the following is irrational?
̅̅̅̅̅
A. 0.125 B. 43.29%
C. √200 D. √343

Author: Victor A. Tondo Jr.


Author: Victor A. Tondo Jr.
Explanation: x = 2x + 2
̅̅̅̅̅ in fraction is
A. 0.125 -2 = 2x - x
-2 = x
B. 43.29% in fraction is
C. √200 cannot be rewritten as a fraction with
whole numbers as numerator and denominator 341. If the difference between the squares of two
D. √343 is 7. consecutive counting numbers is 49, what is the
larger number?
A. 99 B. 49 C. 25 D. 7
338. Aira is six years older than Zayne. Six years
ago, she was twice as old as he. How old is Aira Solution:
now? Let the consecutive counting numbers
A. 21 B. 18 C. 15 D. 12 be x and x + 1.

Solution: (x + 1)2 – x2 = 49
(x2 + 2x + 1) – x2 = 49
Let A Aira’s present age 2x + 1 = 49
A–6 Zayne’s present age 2x = 48
.: A – 6 Aira’s age 6 years ago x =24
A – 12 Zayne’s age 6 years ago .: The larger number is 24 + 1 = 25.
A – 6 = 2 (A – 12)
A – 6 = 2A – 24
342. Rayon needed to find the perimeter of an
-6 + 24 = 2A – A
equilateral triangle whose sides measure x + 4
18 = A
cm each. Jake realized that he could multiply
3 (x + 4) = 3x + 12 to find the total perimeter in
terms of x. Which property did he use to
339. The two parallel sides of a trapezoidal lot
multiply?
measure 100m and 70m. If these sides are 80m
apart, what is the area of the lot? A. Associative Property of Addition
A. 13600 m2 B. 6800 m2 B. Distributive Property of Multiplication over
C. 3400 m2 D. 2400 m2 Addition
C. Commutative Property of Multiplication
Solution: D. Inverse Property of Addition

Area of Trapezoid = 343. A ride in a Feak Taxi costs P25.00 for the
first km and P10.00 for each additional km.
= 80 = 6800 Which of the following could be used to calculate
the total cost, y, of a ride that was x km?
A. y = 25x + 10
340. If y = x and y = 2x + 2, find the value of x. B. y = 10x + 25
A. x = -2 B. x = -1 C. y 25 x 1 + 10
C. x = 0 D. x = 1 D. y 10 x 1 + 25

Solution: 344. Which of the following points is in the


fourth quadrant?
Since y = x and y = 2x + 2, then by transitive
A. (3, 4) B. (-3, 4)
property of equality,
C. (3, -4) D. (-3, -4)

Author: Victor A. Tondo Jr.


Author: Victor A. Tondo Jr.
Explanation: Solution:
2x 3y 5
A point in the fourth quadrant has a positive
2(1) – 3y = 5
abscissa (x-value) and a negative ordinate
2 – 3y = 5
(y-value).
2 – 5 = 3y
-3 = 3y
-1 = y
345. The distance from the sun to the earth is
approximately 9.3 × 107 miles. What is this
distance expressed in standard notation?
349. The sum of two consecutive even integers is
A. 9,300,000,000 B. 930,000,000
126. What is the smaller integer?
C. 93,000,000 D. 651
A. 63 B. 62 B. 61 D. 60

Solution:
346. The square of a number added to 25 equals
10 times the number. What is the number? Let x = smaller even integer
A. -10 B. -5 C. 5 D. 10 .: x + 2 = next even integer

Solution: x + (x + 2) = 126
2x + 2 = 126
x2 + 25 = 10x
2x = 124
x2 – 10x + 25 = 0
x = 62
√ 10 + 25 =√0
x–5=0
x=5 350. Factorize: a2 – a – 90
A. (a – 10) (a + 9)
347. The sum of the square of a number and 12 B. (a + 10) (a + 9)
times the number is 27. What is the smaller C. (a + 10) (a – 9)
possible value of this number? D. (a – 10) (a – 9)
A. -9 B. -3 C. 3 D. 9

Solution: 351. Evaluate 10P5.


A. 2 B. 100,000
Let x = the number C. 1,024 D. 30,240
x2 + 12x = -27
x2
+ 12x + 27 = 0 Solution:
(x + 9) (x + 3) = 0 ! !
10P5 = 30,240
x = -9 or -3 ! !

Or simply use your calculator to evaluate 10P5


348. Let x = 1. Find the corresponding y given by using the nPr button.
that 2x 3y 5.
A. y = -1 B. y = 1
C. y = 3 D. y = -3 352. Jay bought twenty-five P4.57 stamps. How
much did he spend?
A. P 104.25 B. P 114.25
C. P 119.75 D. P124.25

Author: Victor A. Tondo Jr.


Author: Victor A. Tondo Jr.
Solution: They will have the same number of candy bars
sold 3 days after the first day, or on the 4th day.
25 x 4.57 = 114.25

If you’re using a scientific calculator and it shows


356. Which of the following is not a polynomial?
you , simply press then S D button.
A. -3x2 + x – 9 B. √2x + π

C. D. 7 +9
353. Given f(x) = x3 + kx2 – 7, find k if f(2) = 41.
A. 5 B. 10 C. 15 D. 20
Explanation:
Solution: In a polynomial, x cannot be raised to exponents
Just substitute x with 2. that are fractions or irrational numbers.

f(2) = 23 + k(22) – 7
41 = 8 + 4k – 7 357. Factorize: 3p2 – 2p – 5
41 = 4k + 1 A. (3p – 5) (p + 1) B. (3p + 5) (p – 1)
40 = 4k C. (3p + 1) (p – 5) D. (3p – 1) (p + 5)
10 = k

358. The cost of renting a bike at the local bike


354. If y = x and y = 2x + 2, find x + y. shop can be represented by the equation
A. -8 B. -4 C. 0 D. 4 y = 2x + 2, where y is the total cost and x is the
number of hours the bike is rented. Which of the
Solution: following ordered pairs would be possible
Since y = x and y = 2x + 2, then by transitive number of hours rented and the corresponding
property of equality, total cost?
A. 0, 2 B. (2, 6)
x = 2x + 2 C. (6, 2) D. 2, 6
-2 = 2x - x
-2 = x .: y = -2 as well Explanation:
.: x + y = -2 + (-2) = -4 Simply substitute the x- and y-values of the point
into the equation y = 2x + 2. Whichever point
holds a true equation is the correct answer.
355. Mulan and Lilo are competing to see who
can sell the most candy bars for a fundraiser.
Mulan sold 4 candy bars on the first day and 2 359. The distance from the earth to the moon is
each day after that. Lilo sold 7 on the first day approximately 240,000 miles.
and 1 each day after that. On what day will they What is this distance expressed in scientific
have the same number of candy bars sold? notation?
A. 7th B. 6th C. 4th D. 3rd A. 24 × 104 B. 2.4 × 104
C. 2.4 × 105 D. 2.4 × 10 5
Solution:
Let x = number of days to pass after the 1st day 360. =

4 + 2x = 7 + x A. B. C. D.
2x – x = 7 – 4
x=3
Author: Victor A. Tondo Jr.
Author: Victor A. Tondo Jr.
Solution: Solution:

= To find the x-intercept, let y = 0.


3x + 2y = 24  3x + 2(0) = 24
3x = 24
361. The sum of two angles is 180°. The measure x=8
of one angle is 34° greater than the measure of
the other angle. What is the measure of the
364. A circle is drawn such that ̅̅̅̅ is a diameter
smaller angle? and its midpoint is O. Given that C is a point on
the circle, what is the measure of ACB?
A. 74° B. 73° C. 72° D. 71°
A. 180o B. 90o
C. 60o D. not enough info
Solution:
Let x = measure of the smaller angle Explanation:
.: x + 34 = measure of the larger angle
x + (x + 34) = 180 Any inscribed angle that opens
2x + 34 = 180 to a diameter or a semicircle is
2x = 146 a right angle, or measures 90o.
x = 73

362. Solve the following system of linear


equations: 365. Samantha owns a rectangular field that has
3x + y = -9 an area of 3,280 square meters. The length of the
3x 2y 12 field is 2 more than twice the width. What is the
width of the field?
A. 2, 3) B. (2, 3)
A. 40 m B. 41 m
C. (3, 2) D. 3, 2)
C. 82 m D. 84 m
Solution:
Solution:
3x + y = -9
Let w = width of the field
(+) 3x 2y 12
.: 2w + 2 = length of the field
–y = 3
y = -3
w (2w + 2) = 3280
2w2 + 2w = 3280  divide equation by 2
Using the first equation 3x + y 9,
w2 + w = 1640  complete the square
3x + (-3) = -9 w2 + w + 0.25 = 1640 + 0.25
3x = -6 w2 + w + 0.25 = 1640.25
x = -2 √ + + 0.25 √1640.25
w + 0.5 = 40.5
w = 40
363. Find the x-intercept of 3x + 2y = 24
A. x = 8 B. x = -8
C. y = 12 D. y = -12

Author: Victor A. Tondo Jr.


Author: Victor A. Tondo Jr.
366. Rayon used the following mathematical 370. Ten factorial is equal to _____.
statement to show he could change an A. 100
expression and still get the same answer on both B. e10
sides: C. 10 x 9 x 8 x 7 x 6 x 5 x 4 x 3 x 2 x 1
10 × (6 × 5) = (10 × 6) × 5 D. 10e
Which mathematical property did Rayon use?
A. Identity Property of Multiplication
371. How many 3-digit numbers can be made
B. Commutative Property of Multiplication
using the digits 5, 6, 7, 8, 9, and 0 if repetition is
C. Distributive Property of Multiplication over
not allowed?
Addition
D. Associative Property of Multiplication A. 80 B. 100 C. 120 D. 140

Solution:
367. Factorize –x3 27y3.
There are only five possible choices for the
A. (x – 3) (x2 – 3x + 9) hundreds digit since we cannot start with 0.
B. (x – 3) (x2 + 3x + 9) There are five choices for the tens since out of
C. (x + 3) (x2 + 3x – 9) the six, we have used one for the hundreds, and
D. (x + 3) (x2 – 3x + 9) we can use 0 for the tens digit.
We have used two for the hundreds and tens, so
we only have four choices for the units digit.
368. What is the intersection of the lines 5 x 5 x 4 = 100
x + 3y = 5 and -2x + 4y = 0?
A. (2, 4) B. (-2, 1)
C. (2, 1) D. (-2, 4) 372. A researcher is curious about the IQ of
students at the Utrecht University. The entire
Solution:
group of students is an example of a:
2(x + 3y = 5)  2x + 6y = 10 A. parameter B. statistic
-2x + 4y = 0  (+) -2x + 4y = 0 C. population D. sample
10y = 10
y=1 Explanation:
Entirety is population, part of it is sample.
x + 3y = 5  x + 3(1) = 5
x+3=5
x=2 373. Jordan filled a bottle with grains until it was
1/4 full and weighed 8 kg. He added more grains
into the bottle until it was 7/8 full. It now
369. Given f(x) = 7x3 – 3x2 + 2x – 9, f(2) = weighed 18 kg. What is the mass of the empty
A. 56 B. 48 C. 44 D. 39 bottle?
A. 16 B. 8 C. 4 D. 2
Solution:
Solution:
Simply substitute x with 2.
Let B = weight of empty bottle,
f(x) = 7x3 – 3x2 + 2x – 9 G = weight of capacity of bottle
f(2) = 7(23) – 3(22) + 2(2) – 9
f(2) = 56 – 12 + 4 – 9 Eqn 1: B + G = 8
f(2) = 39
Eqn 2: B + G = 18

Author: Victor A. Tondo Jr.


Author: Victor A. Tondo Jr.
Subtract Eqn 1 from Eqn 2: When you get a false equation, it means there is
no solution for the given system of linear
B + G = 18 equations.
(–) B+ G=8
___________________________________________________
Also, when there’s no solution for a system of
G = 10 linear equations, the lines defined by the two
5G = 80 equations are parallel – they will never intersect.
G = 16

Solving for B, 376. Find a and b so that the system below has
the unique solution (-2, 3).
B + ¼ (16) = 8 ax + by = 17
B + 4 = 8; B=4 2ax by 11
374. If 37 – 4x < 17, then
A. x < 5 B. x > 5 A. a 3, b -1 B. a 1, b 5
C. x < -5 D. x > -5 C. a = 1, b = 3 D. a = 1, b = 5

Solution: Solution:
37 – 4x < 17 Substitute x and y with -2 and 3 respectively.
–4x < 17 – 37 ax + by = 17  -2a + 3b = 17
–4x < –20 2ax by 11  -4a – 3b = -11
-¼ (–4x) < -¼ (–20) *
x>5
Solve for a and b by elimination.
*Remember to change your inequality sign after
multiplying or dividing the inequality by a -2(-2a + 3b = 17)  4a – 6b = -34
negative number. -4a – 3b = -11  -4a – 3b = -11

4a – 6b = -34
375. How many solutions are there for the (+) -4a – 3b = -11
following system of linear equations? -9b = -45
-3x + 5y = 6 b=5
6x 10y 0
Using -2a + 3b = 17 and substituting b = 5,
A. only one solution
B. two solutions -2a + 3(5) = 17
C. infinitely many solutions -2a + 15 = 17
D. no solution -2a = 17 – 15
-2a = 2
Explanation: a = -1

Try to solve for x and y first.


-2 (-3x + 5y = 6)  6x 10y = -12 377. What is the probability choosing only one
6x 10y = 0  6x 10y = 0 vowel when three letters are randomly selected
from the word NUMBERS?
6x 10y = -12 A. B. C. D.
(–) 6x 10y 0
0 = -12 which is false.
Solution:
NUMBERS has two vowels and five consonants.
Author: Victor A. Tondo Jr.
Author: Victor A. Tondo Jr.
The number of ways we can get only one vowel Explanation:
out of three is taken as 2C1 x 5C2, or 20.
The equation of the circle is already in the
The number of ways we can get three letters center-radius form. Since r2 = 16, then r = 4.
from seven is 7C3 or 35.
Therefore, the answer is or .
382. A teacher asks students to identity their
favorite reality television show. What type of
measurement scale do the different television
378. If A > B, which is always true? shows make up?
A. B. A2 > B2 A. Nominal B. Ordinal
C. A < B + 2 D. A – B > 0 C. Interval D. Ratio

Explanation:
Use counter-examples to disprove the choices. 383. What is the center of the circle defined by
x2 + y2 – 8x + 6y – 10 = 0?
A. A. (-8, 6) B. (8, -6)
C. (-4, 3) D. (4, -3)
When A = 2 and B = 1, is false.
Explanation:
B. A2 > B2
When A = 2 and B = -3, then 22 > (-3)2 is false. Since the equation of the circle is already in the
form x2 + y2 + Dx + Ey + F = 0, then the center
C. A < B + 2
When A = 1 and B = 0, then 1 < 0 + 2 is false. (h, k) is at ( , or ( , or (4, -3).

D. A – B > 0
This will always be true for any A > B. 384. Find the equation of the line passing (1, 4)
with slope equal to 5.
A. y = 5x + 3 B. y = 5x + 1
379. Statistical techniques that summarize and C. y = 5x – 1 D. y = 5x – 3
organize the data are classified as what?
A. Population statistics Solution:
B. Sample statistics
C. Descriptive statistics Use the form y = mx + b and substitute the x-
D. Inferential statistics and y- values of the point.
x = 1, y = 4, m = 5
380. Five-point Likert scales (strongly disagree, y = mx + b
disagree, neutral, agree, strongly agree) are 4 = 5(1) + b
frequently used to measure motivations and 4–5=b
attitudes. A Likert scale is a: -1 = b
A. Discrete variable.
B. Ordinal variable. y = mx + b
C. Categorical variable. y = 5x + (-1) or y = 5x – 1
D. All of the above

381. What is the radius of the circle defined by


(x + 2)2 + (y – 3)2 = 16?
A. 256 B. 16 C. 8 D. 4
Author: Victor A. Tondo Jr.
Author: Victor A. Tondo Jr.
385. The seminar rooms in the library are 390. Convert 48o to radians.
identified by the letters A to H. A researcher
records the number of classes held in each room A. π rad B. π rad
during the first semester. What kind of graph C. π rad D. π rad
would be appropriate to present the frequency
distributions of these data? Solution:
A. Histogram B. Scatterplot
C. Bar chart D. Box plot To convert from degrees to radians, simply
multiply the given degree measure by °.
386. Find the slope of the line passing the points 48o x =
°
=
A(2, 3) and B(-7, -15). ° °
A. 1 B. -1 C. ½ D. 2

Solution: 391. The median is always:


A. The most frequently occurring score in set of
Use the formula m = data
B. The middle score when results are ranked in
m= = order of magnitude
m=2 C. The same as the average
D. The difference between the maximum and
minimum scores.
387. Factorize: 3n2 – 8n + 4
A. (3n – 2) (n – 2)
B. (3n – 2) (n + 2) 392. A teacher gave a statistics test to a class of
C. (3n + 2) (n + 2) Geography students and computed the measures
D. (3n + 2) (n – 2) of central tendency for the test scores. Which of
the following statements cannot be an accurate
description of the scores?
388. In now many ways can the letters A. The majority of students had scores above the
AAABBCDEEE be arranged in a straight line? mean.
A. 50,400 B. 25,200 B. The majority of students had scores above the
C. 12,600 D. 6,300 median.
C. The majority of students had scores above the
Solution: mode.
! 10! D. All of the above options (A, B and C) are false
50,400 statements.
! ! ! ! 3! 2! 3!
Explanation:
389. Find the smaller angle formed by the x-axis A. If majority of the students had scores above
and the line y = 5x. the mean, then this is an example of a negatively-
A. 78.69o B. 63.48o skewed data. This can happen.
C. 54.15o D. 41.32o B. 50% of the class is always above the median,
and the other 50% of the class is below the
Solution: median.
tan =5 C. The mode can pop up anywhere. That means
= tan-1 5 the students may have a low modal score.
= 78.69o

Author: Victor A. Tondo Jr.


Author: Victor A. Tondo Jr.
393. Find the area of a semicircle whose radius 398. In how many ways can 4 girls and 5 boys be
measures 28 cm. arranged in a row so that all the four girls are
A. 784 π cm2 B. 392 π cm2 together?
C. 28 π cm 2 D. 14 π cm2 A. 4,320 B. 8,640
C. 17,280 D. 34,560
Solution:
A = ½ π r2 Solution:
A = ½ π (28)2 Let 4 girls be one unit. So now, there are 6 units
A = 392 π cm2 in all: 5 boys and the solo unit made by the four
girls. They can be arranged in 6! ways.
394. Find the length of each side of an equilateral In each of these arrangements 4 girls can be
triangle whose perimeter is 90 cm. arranged in 4! ways.
A. 45 cm B. 30 cm ∴ Total number of arrangements in which girls
C. 22.5 cm D. 10 cm are always together
= 6! × 4!
Solution: = 720 × 24
An equilateral triangle has three congruent sides. = 17,280
Its perimeter is given as P = 3s.
3s = 90 399. A box contains 8 batteries, 5 of which are
s = 30 good and the other 3 are defective. Two batteries
are selected at random and inserted into a toy. If
the toy only functions with two good batteries,
395. Find the number of subsets having 4 what is the probability that the toy will function?
elements of the set {1,2,3,4,5,6,7,8,9,10,11}.
A. B. C. D.
A. 165 B. 330
C. 660 D. 1320
Solution:
Solution:
11C4 = 330
400. IQ tests are standardized so that the mean
396. There are ten true - false questions in an score is 100 for the entire group of people who
exam. How many responses are possible? take the test. However, if you select a group of 50
who took the test, you probably would not get
A. 1024 B. 256 C. 20 D. 10 100. What statistical concept explains the
difference between the two means?
Solution:
There are two possible responses per item, and A. Statistical error B. Inferential error
there are ten items. 210 = 1024. C. Residual error D. Sampling error

397. In a 500m speed skating race, time results 401. Which Mathematician pioneered the study
would be considered an example of which level of conic sections?
of measurement? A. Euclid B. Apollonius
C. Archimedes D. Hipparchus
A. Nominal B. Ordinal
C. Interval D. Ratio

Author: Victor A. Tondo Jr.


Author: Victor A. Tondo Jr.
402. A researchers studies the factors that 406. Who is considered by many Mathematicians
determine the number of children future couples as “The Last Universalist”?
decide to have. The variable ‘number of children’ A. Jules Henri Poincare
is a: B. Hendrik Lorentz
A. Discrete variable C. Georg Cantor
B. Continuous variable D. Gottfried Wilhelm Leibniz
C. Categorical variable
D. Ordinal variable Explanation:
Both Poincare and Leibniz are polymaths, but
Explanation:
Jules Henri Poincare is fondly referred to by
Apollonius of Perga was a Greek geometer and fellow Mathematicians as the Last Universalist.
astronomer known for his theories on the topic
of conic sections. His definitions of the terms
ellipse, parabola, and hyperbola are the ones in 407. In the theory he developed, there are
use today. infinite sets of different sizes (called
cardinalities). Which Mathematician formalized
many ideas related to infinity and infinite sets
403. Surface area and volume, center of gravity, during the late 19th and early 20th centuries?
and hydrostatics are some of the studies of A. Jules Henri Poincare
which Mathematician? B. Hendrik Lorentz
A. Apollonius B. Euclid C. Georg Cantor
C. Archimedes D. Hipparchus D. Gottfried Wilhelm Leibniz

408. Which of the following sets of scores has the


404. The book Philosophiæ Naturalis Principia greatest variability or range?
Mathematica, more fondly known simply as A. 2, 5, 8, 11 B. 13, 13, 13, 13
Principia, is the work of which Mathematician? C. 20, 25, 26 ,27 D. 42, 43, 44, 45
A. Euclid B. Newton
C. Einstein D. Archimedes Explanation:
Range = highest score – lowest score
405. A researcher is interested in the travel time
of Rayon’s University students to college. A
group of 50 students is interviewed. Their mean
409. This Mathematician was the first to describe
travel time is 16.7 minutes. For this study, the
a pinwheel calculator in 1685 and invented the
mean of 16.7 minutes is an example of a
wheel named in his honor, which was used in the
A. parameter B. statistic
arithmometer, the first mass-produced
C. population D. sample
mechanical calculator. He also refined the binary
number system, which is the foundation of all
Explanation:
digital computers. Which Mathematician is this,
The 50 students in the above case is only part of who is also crucial to the development of
the population of students of Rayon’s University, computers?
thus, they are only a sample. Data taken from a A. Gottfried Wilhelm Leibniz
sample is called statistic. B. Charles Babbage
C. Ada Lovelace
D. Alexander Graham Bell

Author: Victor A. Tondo Jr.


Author: Victor A. Tondo Jr.
410. Solve for x, given 9x – 10 = 11x + 30 Solution:
A. x = 40 B. x = 20
C. x = -20 D. x = -40

Solution:
9x – 10 = 11x + 30
-10 – 30 = 11x – 9x
-40 = 2x

411. Using Calculus, this Mathematician


explained why tides occur, why the shapes of
415. When five is added to three more than a
planetary orbits are conic sections, and how to
certain number, the result is 29. What is the
get the shape of a rotating body of fluid, among
number?
many other things. Which Mathematician is this?
A. 24 B. 21 C. 8 D. 4
A. Kepler B. Euclid
C. Apollonius D. Newton
Solution:
Explanation: (n + 3) + 5 = 29
n + 8 = 29
Newton explained the above-mentioned topics in
n = 21
his work Principia.

416. The math club is electing new officers.


412. Which of the following terms does NOT
There are 3 candidates for president, 4
describe the number 9?
candidates for vice-president, 4 candidates for
A. rational number B. integer
secretary, and 2 candidates for treasurer. How
C. real number D. prime number
many different combinations of officers are
possible?
Explanation:
A. 13 B. 96
Its factors are 1, 3, and 9. Therefore, 9 is a C. 480 D. 17,160
composite number.
Solution:
Apply the fundamental counting principle.
413. Which expression below is equal to 5?
3 x 4 x 4 x 2 = 96
A. (1 + 2)2 B. 9 – 22
C. 11 10 × 5 D. 45 ÷ 3 × 3
417. Twelve points lie on a circle. How many
cyclic quadrilaterals can be drawn by using these
414. A bus picks up a group of tourists at a hotel.
points? [Note: Cyclic quadrilaterals are
The sightseeing bus travels 2 blocks north, 2
quadrilaterals whose vertices are on a circle.]
blocks east, 1 block south, 2 blocks east, and 1
A. 48 B. 495
block south. Where is the bus in relation to the
C. 11,880 D. 1,663,200
hotel?
A. 2 blocks north B. 1 block west
Solution:
C. 3 blocks south D. 4 blocks east
12C4 = 495

Author: Victor A. Tondo Jr.


Author: Victor A. Tondo Jr.
418. What is the variance for the following set of 422. The sum of five consecutive integers is 215.
scores? 143 143 143 143 143 143 What is the largest of these integers?
A. 43 B. 44 C. 45 D. 46
A. 0 B. 2 C. 4 D. 25
Solution:
Explanation:
Let x = smallest number
The scores are all the same, so the variance is 0.
.: The numbers will be expressed as x, x + 1,
x + 2, x + 3, and x + 4.
419. When 18 is subtracted from six times a x + (x + 1) + (x + 2) + (x + 3) + (x + 4) = 215
certain number, the result is 42. What is the 5x + 10 = 215
number? 5x = 205
A. 10 B. 4 C. -4 D. -10 x = 41

Solution: .: The numbers are 41, 42, 43, 44, and 45.

6x – 18 = -42
6x = -42 + 18 423. You go to the cafeteria for lunch and have a
6x = -24 choice of 4 entrees, 5 sides, 5 drinks, and 4
x = -4 desserts. Assuming you have one of each
category, how many different lunches could be
made?
420. Find the equation of the line passing (2, 3) A. 18 B. 81 C. 40 D. 400
and (-7, -15)
A. y = 2x + 1 B. y = 2x – 1 Solution:
C. y = x + 2 D. y = x – 2
Apply the fundamental counting principle.
Solution: 4 x 5 x 5 x 4 = 400

Use the two-point form.


y – y1 = x x 424. What can be said about the following
statements?
y–3= x 2 i. Any quadrilateral with four congruent sides is
y – 3 = 2(x – 2) a square.
y – 3 = 2x – 4 ii. Any square has four congruent sides.
y = 2x – 1 A. Only the first statement is true.
B. Only the second statement is true.
C. Both statements are true.
421. Of the following Z-score values, which one D. Both statements are fall.
represents the location closest to the mean?
A. Z = +0.5 B. Z = +1.0 Explanation:
C. Z = -1.5 D. Z = -0.3 i. Any quadrilateral with four congruent sides is
a RHOMBUS.
Explanation: ii. Any square has four congruent sides. TRUE.

A lower absolute value of Z-score means the item


is closer to the mean. The Z-score with the
lowest absolute value is D, Z = -0.3.

Author: Victor A. Tondo Jr.


Author: Victor A. Tondo Jr.
425. Out of 6 boys and 4 girls, a committee of 5 429. The second angle of a triangle is three times
has to be formed. In how many ways can this be as large as the first. The measure of the third
done if we take 2 girls and 3 boys? angle is 40 degrees greater than that of the first
A. 120 B. 186 C. 240 D. 256 angle. How large is the first angle?
A. 28o B. 30o C. 35o D. 38o
Solution:
Solution:
6C3 x 4C2 = 20 x 6 = 120
Let x = measure of the first angle
.: 3x = measure of the second angle
426. In the figure, which of the following will x + 40 = measure of the third angle
yield the value of the hypotenuse x?
x + 3x + x + 40 = 180
5x + 40 = 180
5x = 180 – 40 = 140
x = 28

A. x = B. x =
° °
C. x = D. x = 10 tan 35o 430. Normally distributed data are normally
°
referred to as:
A. Bell-shaped B. Asymmetrical
Solution:
C. Skewed D. Peaked
The side measuring 10 units is opposite the
given angle, while x is the hypotenuse. The
trigonometric ratio that involves the opposite 431. A population has a mean of μ 35 and a
side and the hypotenuse is the sine function. standard deviation of σ 5. After 3 points are
added to every score of the population, what are
From the mnemonic SohCahToa, we can recall
the new values for the mean and standard
that sin = . deviation?
A. μ 35 and σ 5
sin 35o = B. μ 35 and σ 8
x sin 35o = 10 C. μ 38 and σ 5
x= D. μ 38 and σ 8
°

Explanation:
427. The shortest side of a 30-60-90 triangle is When a constant is added or subtracted to every
20.19 cm long. How long is the hypotenuse? score of a population, the mean increases or
A. 40.38 cm B. 30.29 cm decreases by the same amount. The standard
C. 34.97 cm D. 17.48 cm deviation, however, stays the same.

Solution:
In a 30-60-90 triangle, the hypotenuse is always 432. Given sin θ = , find cos θ.
twice the shortest side. A. B. C. D.

Author: Victor A. Tondo Jr.


Author: Victor A. Tondo Jr.
Solution: 435. Given cos θ and θ QIV, find tan θ.
Since sin θ = , then θ = sin-1 . A. B. C. D.
Therefore, cos θ = cos (sin-1 )= .
Solution:
Alternative Solution: Since cos θ and cos θ , then
Draw a right triangle containing the angle θ. opposite = √13 5 √144 12.
Mark the side opposite θ as 7 and the
hypotenuse as 25 since sin θ = . Since tan θ , adjacent = 5, and
opposite = 12, by using the CAST mnemonic,
then tan θ since θ QIV.

436. Find the measure of T:


Solve for the missing leg by using the
Pythagorean theorem. You will get the measure
of the adjacent side as √25 7 , or √625 49
which is equal to 24.

A. 59o B. 69o C. 79o D. 89o

437. If the scores on a test have a mean of 26 and


a standard deviation of 4, what is the z-score for
a score of 18?
Now that we know the measure of the adjacent A. -1.41 B. 11 C. -2 D. 2
side, we can substitute cos θ .
Solution:
Z-score =
433. In a triangle, what is located 2/3 of the
distance from each vertex to the midpoint of the Z-score = 2
opposite side?
A. centroid B. incenter
C. circumcenter D. orthocenter 438. If a researcher sets a level of significance at
0.05 (i.e. 5%), what does this mean?
A. Five times out of 100, a significant result will
434. Which triangle has the centroid, incenter,
be found that is due to chance alone and not to
circumcenter, orthocenter, and nine-point-center
true relationship.
at the same location?
B. Ninety-five times out of 100, a significant
A. isosceles right triangle
result will be found that is due to chance alone
B. 30-60-90 triangle
and not to true relationship.
C. equilateral triangle
C. Five times out of 100, a significant result will
D. hyperbolic triangle
be found that is not due to chance, but to true
relationship.
D. None of the above
Author: Victor A. Tondo Jr.
Author: Victor A. Tondo Jr.
Explanation: 443. Although rarely used in proving, what is the
The level of significance is also known as margin extra line or line segment drawn in a figure to
for error. help in a proof?
A. base line B. auxiliary line
C. converse line D. Euler’s line
439. When does a researcher risk a Type I error?
A. Anytime the decision is ‘fail to reject’.
B. Anytime H0 is rejected. 444. What is the measure of V in the following
C. Anytime Ha is rejected. figure?
D. All of the above options

Explanation:
Type I errors happen when we reject a true null
hypothesis.
Type II errors happen when we fail to reject a
false null hypothesis A. 60o B. 65o C. 70o D. 75o

Solution:
440. Solve for x:
90 + 86 + 114 + x = 360
290 + x = 360
x = 70

445. What is the intersection of all three


altitudes of a triangle?
A. incenter B. centroid
A. 4 B. 5 C. 6 D. 7 C. orthocenter D. circumcenter
Solution:
90 + 90 + 98 + 17x – 3 = 360 446. How much water must be evaporated from
275 + 17x = 360 2000 mL of 30% acid solution to make a 50%
17x = 85 acid solution?
x=5 A. 800 mL B. 850 mL
C. 900 mL D. 950 mL

441. Which of the following is equidistant from Solution:


the vertices of the triangle? Let x = amount of water to be evaporated
A. circumcenter B. orthocenter
C. incenter D. centroid C1V1 + C2V2 = CrVr
30%(2000) + 0% (-x) = 50%(2000 – x)
600 + 0 = 1000 – 0.5 x
442. Which of the following is equidistant from
0.5 x = 1000 – 600
the sides of the triangle?
0.5 x = 400
A. circumcenter B. centroid x = 800
C. orthocenter D. incenter
Note: Water is 0% acid, and evaporation means
removal of volume thus –x.

Author: Victor A. Tondo Jr.


Author: Victor A. Tondo Jr.
447. Which of the following is the intersection of 452. In which geometry are there no parallel
angle bisectors of a triangle? lines?
A. circumcenter B. incenter A. elliptic geometry
C. centroid D. orthocenter B. hyperbolic geometry
C. spatial geometry
D. solid geometry
448. In terms of a conditional statement, what is
the statement formed by exchanging and 453. What do we call the ratio of two numbers
negating the antecedent and the consequent? (larger number: smaller number) whose ratio to
A. inverse B. converse each other is equal to the ratio of their sum to
C. adverse D. contrapositive the larger number? [Note: This is applied in
Fibonacci sequences]
Explanation: A. pi B. golden ratio
C. 1.618 D. Euler’s ratio
Statement If p, then q.
Converse If q, then p.
Inverse If not p, then not q. 454. Which of the following pertains to the law of
Contrapositive If not q, then not p. cosines?
A. c2 = a2 + b2 – 2 ab cos C
B. c2 = a2 + b2 + 2 ab cos C
449. What is formed when the hypothesis and C. c2 = a2 + b2 – ab cos C
the conclusion of the conditional statement are D. c2 = a2 + b2 + ab cos C
interchanged?
A. converse B. inverse
C. adverse D. contrapositive 455. Solve for x:

450. What is formed when both the hypothesis


and the conclusion of the conditional statement
are negated?
A. converse B. inverse
C. adverse D. contrapositive
A. x = 8 B. x = 9
C. x = 10 D. x = 11
451. Which of the following is the converse of the
following statement?
456. What do we call an angle formed by two
“If two angles are congruent, chords of the circle with a common endpoint
then they have the same measure.” (the vertex of the angle)?
A. If two angles are not congruent, then they do A. inscribed angle
not have the same measure. B. tangential angle
B. If two angles have the same measure, then C. circumscribed angle
they are congruent. D. interior angle
C. If two angles do not have the same measure,
then they are not congruent.
D. If two angles are not congruent, then they
have the same measure.”

Author: Victor A. Tondo Jr.


Author: Victor A. Tondo Jr.
457. Find the inverse of y = . 461. What do we call three positive integers with
the property that the sum of the squares of two
A. y-1 = B. y-1 = of the integers equals the square of the third?
C. y-1 = D. y-1 = A. Euclid’s triple
B. Pythagorean triple
Solution: C. Newton’s triple
D. Cartesian triple
First, solve for x in terms of y.
y=
462. Which of the following expressions will give
2020 y = 2018 x + 2019 the value of x?
2020 y – 2019 = 2018 x
=x

Then switch y and x.


=y
A. 50 tan 37o B. 50 cos 37o
Lastly, replace y with y-1. C. 50 sin 37o D. 50 cot 37o

= y-1. Solution:
tan 37o =
457. In ∆ABC, m A = 58o, and m B = 74o. Find 50 tan 37o = x
m C.
A. 38o B. 43o C. 48o D. 53o
463. Solve for x:
Solution:
58 + 74 + x = 180
132 + x = 180
x = 48

458. Which lines are not in the same plane and


do not intersect but are not parallel?
A. asymptotes B. tangent lines A. 6 B. 7 C. 7.5 D. 8
C. skew lines D. directrices
Solution:
94 + 86 + 94 + 11x – 2 = 360
459. Two adjacent angles whose distinct sides lie 272 + 11x = 360
on the same line are called what? 11x = 360 – 72 = 88
A. linear pair B. vertical pair x=8
C. alternate D. corresponding

464. A bus travels 600 km in 7 hrs and another


460. The point of concurrency of a triangle’s 300 km in 5 hrs. What is its average speed?
three altitudes is called _____. A. 72.86 kph B. 75 kph
A. circumcenter B. incenter C. 77.86 kph D. 80 kph
C. orthocenter D. centroid

Author: Victor A. Tondo Jr.


Author: Victor A. Tondo Jr.
Solution: 467. How many ways can the word PILIPINAS be
rearranged?
Average speed = total distance / total time
A. 302 B. 3,024
Average speed = (600 + 300) / (7 + 5)
C. 30,240 D. 302,400
Average speed = 900/12 = 75 kph
Solution:
465. A sniper on a cliff observes that the angle of This problem is on permutation with repeated
depression to his target is 30o. If the cliff is 10 elements. PILIPINAS has nine letters, of which
meters high, how far must the bullet travel to hit two are P, three are I, and the other letters are
the sniper’s target? singular.
A. 20 meters B. 10√3 meters 9!
30,240
C. 10√2 meters D. 10 meters 2! 3!

Solution:
468. A coin is tossed 60 times. Head appeared 27
times. Find the experimental probability of
getting heads.
A. B. C. D.

Explanation:
Let trajectory = x The experimental probability of an event is the
sin 30o = ratio of the number of times the event occurs to
the total number of trials.
0.5 =
x= = 20
.
469. A parabola is defined by the equation
5x = -3y2 – 4y + 2. Which of the following is true
466. Rowena received a total of 25 bills. These about the parabola?
bills are either P20 or P50 bills. If Rowena A. It opens to the left.
received an amount of P800, how many P20 bills B. It opens to the right.
did she receive? C. It opens upward.
A. 10 B. 13 C. 15 D. 17 D. It opens downward.

Solution: Explanation:

Let x = number of P20 bills The squared variable is y and the coefficient of y2
25 – x = number of P50 bills is negative. Therefore it opens to the left.

20(x) + 50(25 – x) = 800


20x + 1250 – 50x = 800 470. Find the equation of the circle whose center
1250 – 800 = 50x – 20x is at (7, -24) given that it passes the point of
450 = 30x origin.
15 = x A. (x + 7)2 – (y – 24)2 = 961
B. (x – 7)2 + (y + 24)2 = 961
C. (x + 7)2 + (y – 24)2 = 625
D. (x – 7)2 + (y + 24)2 = 625

Author: Victor A. Tondo Jr.


Author: Victor A. Tondo Jr.
Explanation: 474. The shortest leg of a 30-60-90 triangle is
123 cm long. How long is the leg adjacent to the
Use the center-radius form. The radius is the
30o angle?
distance from the center (7, -24) and the point of
origin (0, 0). A. 123 cm B. 123√2 cm
C. 123√3 cm D. 246 cm

471. Ana left their house and jogged at a speed of Explanation:


60 meters per minute. Bea followed her two The leg adjacent to the 30o angle in a 30-60-90
minutes later and jogged at a speed of 70 meters triangle is always √3 times the length of the
per minute. How many minutes after Bea left shortest leg.
would she catch up with Ana?
A. 14 B. 13.5 C. 13 D. 12
475. How many odd 4digit numbers can be
Solution: formed using the digits 7, 6, 5, 4, 3, 2, and 1 if
Let x = number of minutes after Bea left to catch repetition is not allowed?
up with Ana A. 720 B. 480 C. 240 D. 120
60 (x + 2) = 70 (x)
Solution:
60x + 120 = 70x
6 x 5 x 4 x 4 = 480
120 = 70x – 60x
120 = 10x
12 = x
476. There are 70 dogs and geese in a farm. If
there are a total of 200 legs, how many dogs are
there?
472. How many mL of 40% acid must be added
A. 25 B. 30 C. 35 D. 40
to 1000 mL of 10% acid solution to make a 20%
acid solution? Solution:
A. 250 B. 500 C. 600 D. 750
Leg D = number of dogs
Solution: G = number of geese
Let x = amount to be added in mL D + G = 70  2D + 2G = 140
4D + 2G = 200  4D + 2G = 200
40 (x) + 10 (1000) = 20 (x + 1000)
-2D = -60
40x + 10,000 = 20x + 20,000
D = 30
40x – 20x = 20,000 – 10,000
20x = 10,000
x = 500
477. Find the maximum area of a rectangle if the
perimeter is set at 350 cm.
A. 8656.25 cm2 B. 7656.25 cm2
473. The hypotenuse of a 30-60-90 triangle is
C. 6656.25 cm 2 D. 5656.25 cm2
432 cm long. How long is the leg opposite the 30o
angle? Shortcut:
A. 216 cm B. 216 √2 cm
C. 216 √3 cm D. 432 cm Make it a square.
P = 4S = 350
Explanation: S = 87.5 .: A = S2 = 87.52
A = 7656.25
The shorter leg in a 30-60-90 triangle is always
half the length of the hypotenuse.
Author: Victor A. Tondo Jr.
Author: Victor A. Tondo Jr.
478. A rectangle is 60 cm long and 45 cm wide. Explanation:
How long is its diagonal?
This problem is on circular permutations so use
A. 75 cm B. 85 cm
(n – 1)!.
C. 95 cm D. 105 cm

Solution:
483. There are 24 mangoes in a basket, of which
The diagonal D is given as √ + . 7 are rotten. What is the probability that when
randomly getting two mangoes at the same time,
√45 + 60 √2025 + 3600 both are rotten?
D = √5625 = 75 A. B. C. D.

Solution:
479. Find the length of the diagonal of a cube
given each side measures 17 cm.
A. √290 cm B. 17√2 cm
C. 17√3 cm D. 34 cm
484. In a gathering of gamers and admins, there
Solution: are 24 gamers of which 6 are females, and 3
admins of which one is female. If a female is
In a cube, the diagonal D is √3 times the measure
randomly called, what is the probability that she
of each side.
is an admin?
A. B. C. D.
480. Find the measure of each exterior angle of a
regular 20-sided polygon. Explanation:
A. 162o B. 150o
There are only seven females, of which one is an
C. 144o D. 126o
admin.
Explanation:
MEA = 180 - where n = number of sides of 485. What is the remainder when 3x6+ 4x5 – 5x4
+ 6x3 + 7x2 – 8x + 3 is divided by (x – 1)?
regular polygon
A. 8 B. 9 C. 10 D. 11

Solution:
481. How many diagonals does a regular 14-
sided polygon have? Use the Remainder Theorem:
A. 81 B. 91 C. 96 D. 101 3(1)6+ 4(1)5 – 5(1)4 + 6(1)3 + 7(1)2 – 8(1) + 3
= 3 + 4 – 5 + 6 + 7 – 8 + 3 = 10
Solution:

D= 91 486. Seven people have an average weight of 49


kg. A child was added to the group and the
average became 45 kg. How heavy is the child?
482. How many ways can 14 people be seated in A. 15 kg B. 16 kg
a Ferris wheel given that each cart can only C. 17 kg D. 18 kg
contain one person?
A. 15! B. 14! C. 13! D. 12!

Author: Victor A. Tondo Jr.


Author: Victor A. Tondo Jr.
Solution: Solution:
Let N child’s weight The diameter of the sphere is of the same length
as each side of the cube. That means the
45 = diameter of the sphere is 20 cm, and
45 subsequently, the radius is 10 cm.
360 = 343 + N SA = 4 r2
17 = N SA = 4 (102) = 400 cm2

487. Six numbers have an average of 71. If 85 is 491. Given f(x) = (25 x20 – 24x10)(x2 – 9x + 3),
added to the group, what is the new average? find f ’ x .
A. 72 B. 73 C. 74 D. 75 A. f ‘ x 550 x21 – 4725 x20 + 1500 x19 – 288 x11
+ 2376 x10 – 720 x9
Solution: B. f ‘ x 450 x21 – 4725 x20 + 150 x19 + 288 x11
+ 2376 x10 – 720 x9
Sum of the first six numbers: 6(71) = 426
C. f ‘ x 550 x21 – 4725 x20 + 150 x19 – 288 x11
New average = 73 + 2376 x10 – 720 x9
D. f ‘ x 450 x21 – 4725 x20 + 1500 x19 – 288 x11
+ 2376 x10 – 720 x9
488. In an arithmetic sequence, the 7th term is 25
and the 10th term is 67. What is the common Solution:
difference?
A. 42 B. 21 C. 14 D. 7 Use the product rule.
Let u = (25 x20 – 24x10)
Solution: and v = (x2 – 9x + 3)
D= 14 .: du = 500 x19 – 240 x9
dv = 2x – 9
f‘ x = u dv + v du
489. Triangle ABC has sides measuring 20 cm, 20 = (25 x20 – 24x10) (2x – 9) + (x2 – 9x + 3)
cm, and 29 cm. What kind of triangle is ABC? (500 x19 – 240 x9)
A. acute B. right = 50 x21 – 48 x11 – 225 x20 + 216 x10 +
C. obtuse D. reflex 500 x21 – 4500 x20 + 1500 x19 – 240 x11 +
2160 x10 – 720 x9
Explanation: = 550 x21 – 4725 x20 + 1500 x19 – 288 x11
For any triangle with sides X, Y, and Z, given + 2376 x10 – 720 x9
that X Y Z, if X2 + Y2 > Z2, then the triangle is
obtuse. Alternative Solution:
f(x) = (25 x20 – 24x10)(x2 – 9x + 3)
= 25x22 – 225x21 + 75x20 – 24x12 + 216 x11
490. Find the surface area of a sphere given that – 72 x10
the sphere sits perfectly inside a cube whose
f‘ x 550 x21 – 4725 x20 + 1500 x19 – 288 x11
sides measure 20 cm each.
+ 2376 x10 – 720 x9
A. 400 cm2 B. 800 cm2
C. 1200 cm 2 D. 2400 cm2

Author: Victor A. Tondo Jr.


Author: Victor A. Tondo Jr.
492. Simplify: eln 2019 x Solution:
A. 2019x B. 2019 x
7C2 x 6C2 = 21 x 15 = 315
C. x D.

Explanation: 496. Find the measure of the smaller angle


formed by the hands of the clock at 11:20.
Remember that eln u = u.
A. 130o B. 135o
In the question, u = 2019 x. Therefore,
C. 140o D. 145o
eln 2019 x = 2019 x
Solution:
493. If the roots of a quadratic equation are A = 30h - m = 30(11) - (20)
and , which of the following could be the = 330 – 110
= 220
quadratic equation?
A. 63x2 + 22x – 21 = 0 Since the angle taken from our formula is a reflex
B. 63x2 – 22x – 21 = 0 angle and we are looking for the smaller angle,
C. 63x2 + 22x + 21 = 0 then 360 – 220 = 140.
D. 63x2 – 22x + 21 = 0

Solution: 497. How many even 3-digit even numbers can


x= x= be formed using the digits 7, 6, 5, 4, 3, 2, 1, and 0
9x = -7 7x = 3 if repetition is not allowed?
9x + 7 = 0 7x – 3 = 0 A. 150 B. 160 C. 170 D. 180

(9x + 7) (7x – 3) = 0 Solution:


63x2 + 49x – 27x – 21 = 0
63x2 + 22x – 21 = 0 Number of 3-digit numbers:
7 x 7 x 6 = 294
Number of 3-digit odd numbers:
494. If three more than twice a number is 6 x 6 x 4 = 144
seventeen less than seven times the number, .: Number of 3-digit even numbers:
what is the number? 294 – 144 = 150
A. 2 B. 3 C. 4 D. 5

Solution: 498. There are 50 students in a class. Twenty of


them have a laptop. Thirty-two of them have a
2x + 3 = 7x – 17 smartphone. Seven of them have both a laptop
3 + 17 = 7x – 2x and a smartphone. How many of them have
20 = 5x neither a laptop nor a smartphone?
4=x A. 4 B. 5 C. 6 D. 7

Solution:
495. A team is to be made from a group of seven 50 – (20 + 32 – 7) = 50 – 45 = 5
teachers and six scientists. If the team is to be
composed two teachers and two scientists, how
many different ways can they form a team?
A. 325 B. 315 C. 300 D. 285

Author: Victor A. Tondo Jr.


Author: Victor A. Tondo Jr.
499. Mocha can finish a job in 24 hours, while
her sister Tiramisu can do the same job in only
20 hours. How long will it take them to finish the
job by working together?
A. hrs B. hrs
C. 11 hrs D. hrs

Solution:
hrs

500. What conic figure does the equation


x2 + y2 + 10x – 16y = -100 form?
A. Real circle B. Degenerate circle
C. Imaginary circle D. Ellipse

Solution:
Use CTS (completing trinomial squares) to
convert the equation to its center-radius form.
x2 + y2 + 10x – 16y = -100
x2 + 10x + y2 – 16y = -100
(x2 + 10x + 25) + (y2 – 16y + 64) = -100 + 25 + 64
(x + 5)2 + (y – 8)2 = -11

Since r2 = -11, then the radius is imaginary,


making it an imaginary circle.

Questions and Solutions by:

Victor A. Tondo Jr., LPT


Admin, LET Review for Math Majors

Author: Victor A. Tondo Jr.

You might also like